Categories
Distribution Exam Questions Harvard Theory

Harvard. Final exam for distribution theory. Carver, 1905-1906

 

Thomas Nixon Carver was hired by Harvard based on his work in economic theory. As it turned out theory would only constitute a minor share of his portfolio of courses at Harvard. Here we have the exam for a theoretical course offered in 1905-06 dedicated to the functional distribution of income. This is the second time Thomas Nixon Carver taught this one-semester course at Harvard. (Exam from its initial run in 1904-05).

The course content is undoubtedly captured in Carver’s 1904 book The Distribution of Wealth which was reprinted several times during his lifetime.

__________________________

Course Enrollment
Distribution of Wealth
1905-06

Economics 14a 1hf. Professor Carver. — The Distribution of Wealth.

Total 46: 7 Graduates, 25 Seniors, 9 Juniors, 2 Sophomores, 3 Others.

Source: Harvard University. Report of the President of Harvard College, 1905-1906, p. 73.

*  *  *  *  *  *  *  *  *  *  *  *  *  *  *  *

ECONOMICS 14a
Distribution of Wealth
Year-end Examination, 1904-05

  1. Why does the value of a consumable commodity fall when its supply increases, — other things remaining the same?
  2. Ditto of a productive factor?
  3. What limits or checks the supply of labor?
  4. Ditto of land?
  5. Ditto of capital?
  6. Fill out the blank columns in the [following] table.

Total Crop, Marginal Product of Labor, Total Wages, and Rent (All in Bushels) from Four Farms of Different Productivity when cultivated by Varying Numbers of Laborers, Capital being left out of account.

Farm A

Total crop Marginal product Total wages Total rent

Rent per acre

1 500
2 900
3 1200
4 1400
5 1500
 

Farm B

Total crop Marginal product Total wages Total rent

Rent per acre

1 400
2 700
3 900
4 1000
5
 

Farm C

Total crop Marginal product Total wages Total rent

Rent per acre

1 300
2 500
3 600
4
5
 

Farm D

Total crop Marginal product Total wages Total rent

Rent per acre

1 200
2 300
3
4
5
  1. In what proportion could six laborers be most advantageously distributed among these farms? Ten laborers? Fourteen laborers?
  2. When there are six laborers employed, how much, in bushels, would the product of the whole group of farms and laborers be reduced by the removal of one laborer, assuming the laborers all to be of the same efficiency? Ditto when there are ten laborers?
  3. When there are fourteen laborers employed on these farms, how much, in bushels, would the product of the whole group be increased by the opening up of a new farm of the same grade as farm A, and the transfer to it of four of the laborers?
  4. Compare Clark’s theory of business profits with Walker’s.
  5. State Hollander’s position on the question. Does rent enter into price?
  6. Compare Clark’s definition of capital with Taussig’s.

Source: Harvard University Archives. Harvard University. Mid-year Examinations, 1852-1943. Box 7, Bound Volume: Examination Papers, Mid-Years 1905-06.

Image Source: Portrait of Thomas Nixon Carver from the Harvard Class Album 1913. Colorized by Economics in the Rear-view Mirror.

 

Categories
Agricultural Economics Exam Questions Fields Harvard History of Economics Industrial Organization Money and Banking Public Finance Sociology Theory Undergraduate

Harvard. Division Exams for A.B., General and Economics, 1921

The Harvard Economics department was once one of three in its Division in the Faculty of Arts and Sciences. The Departments of History and Government shared a general division exam with the Department of Economics and also contributed their own specific exams for their respective departmental fields. This post provides the questions for the common, i.e. general, divisional exam, the general economics exam, and all the specific exams at the end of the academic year 1920-21 for those fields falling within the perview of the economics department.

_______________________________

Previously posted
Division A.B. Exams

Division Exams 1916
Division Exams, January 1917
Division Exams, April 1918
Division Exams, May 1919
Division Exams, April/May 1920

Division Exams 1931

Special Exam for Money and Government Finance, 1939
Special Exam Economic History Since 1750, 1939
Special Exam for Economic Theory, 1939
Special Exam for Labor and Social Reform, 1939

_______________________________

DIVISION OF HISTORY, GOVERNMENT AND ECONOMICS

EXAMINATIONS FOR THE DEGREE OF A.B.
1920-21

DIVISION GENERAL EXAMINATION

PART I

The treatment of one of the following questions will be regarded as equivalent to one-half of this examination and should therefore occupy one hour. Write on one question only. Insert before your answer to this question a sketch of your plan of treatment.

  1. Discuss the relations of civilization to climate.
  2. Does history show that the periods of a nation’s political and literary greatness tend to coincide?
  3. Was America’s entrance into the World War a consequence or a violation of her policies and traditions?
  4. Discuss the following: “One of the great difficulties, as well as one of the great fascinations of history is the constantly changing point of view; but we should beware of interpreting the past in the light of the present.”
  5. What have been and what should be the limitations upon the application of the principle of self-determination in national relations?
  6. Contrast Roman provincial, and nineteenth-century colonial relations.
  7. What should be the limits of nationalization of essential industries?
  8. What have been the marked characteristics of three great states at the time of their greatest power?
  9. “Society has departed very widely from the strict rule of non-interference with industry by the State; indeed, the policy of non-interference was never carried out logically by any State.” Comment.
  10. Discuss: “The patriotism of nations ought to be selfish.”
  11. What are the standards of social justice?

PART II

The treatment of four of the following questions in Part II is required and will be regarded as equivalent to one-half of this examination, and should therefore occupy one hour. The four questions are to be taken from the Departments in which the student is NOT CONCENTRATING; two questions from each of the two Departments.

A. HISTORY

  1. Briefly characterize, with approximate dates, five of the following: Alexander, Aristotle, Augustus, Francis Bacon, Frederick Barbarossa, Bolivar, Calvin, Chatham, Franklin, Richelieu.
  2. Give a short account of the rise of the Christian Church down to the period of the Crusades.
  3. Estimate the importance of the Netherlands in the development of Europe.
  4. Discuss the relations of England and the United States during the past one hundred years.
  5. Write a brief historical account of slavery in the Western Hemisphere.

B. GOVERNMENT

  1. Discuss: “Not independence but interdependence is the hope of nations.”
  2. Explain the evolution and significance of trial by jury.
  3. What is the significance of the following headlines in March, 1921?
    1. “Austria in dangerous unrest.”
    2. “Briand voted confidence on reparations.”
    3. “Crown prince is plotting.”
    4. “Lenin knows his Italian friends.”
  4. What are the limits of uniform state legislation?
  5. What political unities can best control:
    1. police,
    2. water supply,
    3. roads?

C. ECONOMICS

  1. “The fundamental fact in history is the law of decreasing returns. It is the cause of the origin and development of civilization. . . . It is equally, and for the same reason, the source of poverty and war.”
    State, explain, and indicate the significance of the law of decreasing (diminishing) returns.
  2. What are the fundamental features of the organization of modern industrial society?
  3. Discuss one of the following statements:
    1. “Employees have the right to contract for their services in a collective capacity, but any contract that contains a stipulation that employment should be denied to men not parties to the contract is an invasion of the constitutional rights of the American workmen, is against public policy, and is in violation of the conspiracy laws.”
    2. “In the old days, America outsailed the world. . . . I want to acclaim the day when America is the most eminent of shipping nations. . . . A big navy and a big mercantile marine are necessary to the future of the country.”
  4. Why should there be a labor party in England and not in the United States?
  5. What are the economic essentials of socialism?

_______________________________

GENERAL EXAMINATION
DEPARTMENT OF ECONOMICS

I

The treatment of two of the following questions will be regarded as equivalent to one-half of the examination and should therefore occupy one hour. Write on two questions only.

  1. Give the author, approximate date, and general character of five of the following works:
    1. National System of Political Economy.
    2. Essays in Political Arithmetick.
    3. England’s Treasure by Forraign Trade.
    4. Essay on the Principle of Population
    5. Principles of Political Economy.
    6. The Wealth of Nations.
    7. Das Kapital.
    8. Lombard Street.
    9. Capital and Interest.
  2. Explain four of the following terms:
    Abstinence; Manchester School; stationary state; iron law of wages; produit net; non-competing groups; Scholasticism; Utilitarianism.
  3. Locate on an outline map:
    1. The world’s principal sources of five of the following raw materials: cotton; copper; sugar; silk; wheat; tin; rice; nitrate; petroleum; gold.
    2. The more important routes of overseas transportation.
    3. The world’s chief regions of manufacture.

II

The treatment of three of the following questions will be regarded as equivalent to one-half of the examination and should therefore occupy one hour. Write on three questions only. Be concise.

  1. Define “thrift” and discuss its social significance.
  2. Analyze the determination of normal value under competitive conditions of joint cost.
  3. What is meant by “monetary inflation”? How is it to be measured and what is its importance?
  4. What has been the course of the interest rate in modern times? What probably will be the course of the rate during the next few years? Why?
  5. What are the purposes and limits of progressive taxation?
  6. Discuss the future of public utilities in the United States.
  7. To what extent and in what respects, if at all, is labor legislation of the times a corrective of the more serious defects of the existing social order?
  8. Discuss: “Perpetual prosperity would be a national calamity.”

_______________________________

SPECIAL EXAMINATION
ECONOMIC THEORY

Answer six questions

A

Take from this group at least two and not more than four

  1. What is the concept of “justice” in the theory of the distribution of wealth?
  2. Comment on the validity and significance of the following contention: “Labor is the source of all wealth.”
  3. “Whether capital is productive depends simply on the question: Are tools useful? It matters not how much or how little tools add to the product — if they add something, capital is productive.” Do you agree? Explain.
  4. “The forces which make for Increasing Return are not of the same order as those that make for Diminishing Return. . . . The two ‘laws’ are in no sense coordinate. . . . The two ‘laws’ hold united, not divided, sway over industry.” Comment critically.
  5. What relations exist between the accounting and economic concepts of “cost of production”?
  6. “The differences in the productive powers of men due to their heredity or social position give to certain individuals the same kind of an advantage over others that the owner of a corner lot in the center of a city has over one in the suburbs. If the income from a corner lot is a surplus and can therefore be described as unearned, the income of a man of better heredity, education or opportunity must also be regarded as a surplus income and therefore unearned.”
    Discuss this statement with reference to your general theory of distribution.

B

Take from this group at least one and not more than wto

  1. Give a brief historical account of the theory of population.
  2. Trace the development of the theory of international trade.
  3. In what ways have the following influenced the history of economic thought: Aristotle, Thomas Aquinas, Malthus, Ricardo, J.S. Mill, Marx?
  4. Outline the evolution of the theory of economic rent.

C

Take from this group at least one and not more than two

  1. “The profits of speculation on the Stock Exchange are just as unearned as the increment in the value of urban building sites; unlike the profits of speculation in produce, they represent no service to society.” Do you agree? Why, or why not?
  2. “There is a point beyond which advertising outlay is extravagant.” Explain.
  3. “I do not see how we can retain our home markets, upon which American good fortune must be founded, and at the same time maintain American standards of production and American standards of living unless we make other peoples with lower standards pay for the privilege of trading in the American markets.” Discuss.
  4. What are the advantages and disadvantages of the closed shop?

_______________________________

DIVISION SPECIAL EXAMINATION
ECONOMIC HISTORY

Answer six questions 

A

Take from this group at least one and not more than two

  1. “The opening of the Erie Canal affected both intensive and extensive agriculture in the United States.” Explain. Have there been analogous changes in later periods?
  2. Discuss the following statement: “The enactment of corporation laws by the various states is the most important step made during the past century in the development of American manufactures.”
  3. Analyze the important economic after-effects of the World War.
  4. Briefly explain the most satisfactory methods for separating the different types of variation in time series.

B

Take from this group at least two and not more than four

  1. Write a brief account of one of the early English trading companies.
  2. Sketch the rise of the modern factory system.
  3. Compare changes in farm ownership and tenancy during the nineteenth century in England and the United States.
  4. Outline the history of banking in the United States from 1830 to 1860.
  5. Write a brief narrative of the early development of the railroad.
  6. Give the history of the Sherman Silver Purchase Act.
  7. Trace the evolution of the middle class and forecast its future.

C

Take from this group at least one and not more than two

  1. Give a critical account of the policy of the Federal Government in its regulation of industrial combinations.
  2. Discuss the history and consequences of immigration into the United States since 1840.
  3. Review the development of German foreign trade before the War with special reference to the methods of trade promotion.
  4. Analyze the causes, extent, and consequences of changes in the price level in the United States since 1914.

_______________________________

DIVISION SPECIAL EXAMINATION
PUBLIC FINANCE

Answer six questions

A

Take from this group at least one and not more than two

  1. A law of 1691 authorizes the municipal corporations of New York “to impose any reasonable tax upon all houses within said city, in proportion to the benefit they shall receive thereby.” How far is this a correct principle of taxation and how far has it continued to be applied?
  2. Present a classification of Federal expenditures for a national budget system.
  3. Give a brief account of the financial statistics issued currently by the Federal Government.
  4. Discuss the proposal for the cancellation of all inter-allied debts.

B

Take from this group at least one and not more than two

  1. How has the Federal Constitution influenced national and state tax systems in the United States?
  2. Trace the history of an important fiscal monopoly.
  3. Give a brief account of the financial history of one of the American states.
  4. What connections have existed between currency systems and government finance? Illustrate fully.

C

Take from this group at least two and not more than four

  1. Compare the total expenditures in the United States in normal times for (a) national, (b) state, and (c) municipal purposes. What changes, if any, in the proportions are to be expected?
  2. To what extent is it desirable to separate state and local revenues in the United States?
  3. Indicate the nature and significance of the “grant in aid” in British public finance.
  4. What arguments have been used in European countries for and against a capital levy?
  5. Should the poll tax be abolished? Why, or why not?
  6. Discuss critically the present condition of the public debt of the United States.

_______________________________

DIVISION SPECIAL EXAMINATION
MONEY AND BANKING

Answer six questions

A

Take from this group at least one and not more than two

  1. What part, if any, do commercial banks play in (a) the process of investment; (b) the increase of capital; (c) the course of industrial development; (d) leadership in the business world? In what respects, if at all, may the influence of commercial banks be economically inexpedient?
  2. Discuss the desirability of uniform bank accounting in the United States.
  3. Describe critically the more important sources of statistics of currency and credit in the United States.
  4. Analyze the successive phases of the business cycle. What are the causes of financial panics; industrial crises?

B

Take from this group at least one and not more than two

  1. Give a brief account of the life and work of John Law.
  2. Trace the history of usury laws.
  3. Outline the political background of American monetary history from 1870 to 1900.
  4. Give a brief history of the Reichsbank.

C

Take from this group at least two and not more than four

  1. “It is quite clear that the money question no longer survives as a political issue.” Do you agree? Why, or why not?
  2. To what extent has the status of the gold standard been affected by the World War?
  3. “This little neutral country [Switzerland], surrounded by four great continental belligerents, and bordering on the two principal battle-fronts of Europe, possesses at present, curiously enough, an exceptional purchasing power. This is the consequence of the high level of Swiss currency, which is 250 per cent above the usual parity with the currency of the neighbor in the east, Austria-Hungary; 100 per cent higher than that of the neighbor in the north, Germany; 90 per cent higher than that of the neighbor in the south, Italy; and 20 per cent higher than that of the western neighbor, France. Even in overseas countries, Swiss currency has a higher buying power than the English sovereign or the American dollar.” Explain fully.
  4. What changes have been made in the original Federal Reserve System? What have been the purposes and effects of the changes? What further changes, if any, seem desirable?
  5. Compare the provisions for agricultural credit in two important countries.
  6. Comment upon the following statement: “Prosperity continued through the war, and gave the nation such a tremendous start in business activity that we would still be rejoicing in a period of great prosperity had it not been for the death-dealing blow of deflation of credit given by Mr. Wilson’s Federal Reserve Board.”

_______________________________

DIVISION SPECIAL EXAMINATION
CORPORATE ORGANIZATION, INCLUDING RAILROADS

Answer six questions

 A

Take from this group at least one and not more than two

  1. State the theory of value under conditions of monopoly. In what ways, if at all, is monopoly price affected by (a) cost of production per unit; (b) potential competition; (c) an elastic demand for the product; (d) the existence of satisfactory substitutes for the product; (e) hostile public opinion?
  2. Formulate a statistical classification of business organizations in the United States.
  3. Discuss the apportionment of railway operating expenses between freight and passenger service.
  4. Analyze the valuation of corporate assets from the standpoint of the principles of accounting.

B

Take from this group at least one and not more than two

  1. Compare the history of business corporations in England and the United States.
  2. Trace connections between railroad construction in the United States and related political and economic events.
  3. Give a brief narrative of the trust dissolutions of the Federal Government.
  4. What provisions of the Federal Constitution have been most important in determining policies of government regulation of public utilities?

C

Take from this group at least two and not more than four

  1. Discuss the following statement: “The enactment of corporation laws by the various states is the most important step made during the past century in the development of American manufactures.”
  2. What are the advantages and disadvantages of non-par stock?
  3. Discuss the probable consequences of the Supreme Court decision that stock dividends are not income under the income tax law.
  4. What is the nature and importance of good-will in corporation finance?
  5. To what extent may there be differences in the fair valuation of public utilities for the purposes of rate-making, condemnation, taxation, and capitalization?
  6. Did the Government act wisely in returning the railroads March 1, 1920 to their corporate owners for operation? Why, or why not?

_______________________________

DIVISION SPECIAL EXAMINATION
ECONOMICS OF AGRICULTURE

Answer six questions 

A

Take from this group at least one and not more than two

  1. Analyze the doctrine of economic rent from agricultural land.
  2. What are the functions of organized speculation in staple agricultural products?
  3. Describe the methods to be employed in making an annual farm inventory.
  4. What subjects are covered by the decennial Federal census of agriculture? What is the statistical value of the results of the several inquiries?

B

Take from this group at least one and not more than two

  1. Trace the history of the relations between landlords and tenants in England.
  2. What have been the most important changes in American agriculture since 1890?
  3. Give a critical account of the land policies of the Federal Government.
  4. Outline the development of the beet sugar industry in Europe.

C

Take from this group at least two and not more than four

  1. What factors determine the most efficient size of farms?
  2. What are the advantages of diversification of crops?
  3. Discuss the future of the meat supply of the United States.
  4. Describe and estimate the advantages and disadvantages of the different methods of marketing farm produce.
  5. State and defend a forest conservation policy for the United States.
  6. Compare the provisions for agricultural credit in two important countries.
  7. What are the principal problems of rural community life in the United States?

_______________________________

DIVISION SPECIAL EXAMINATION
LABOR PROBLEMS

Answer six questions

 A

Take from this group at least one and not more than two

  1. Discuss the proposal to restrict immigration into the United States by limiting the number of each nationality admitted each year to 3 per cent of the foreign-born of that nationality resident in this country in 1910.
  2. Describe the technique of statistical measurement of the high cost of living.
  3. What are the principal difficulties encountered in the collection of wage statistics?
  4. Analyze the relations between high money wages and high commodity prices.

B

Take from this group at least one and not more than two

  1. Describe the early development of the factory system.
  2. Trace the origins of trade-unionism in the United States.
  3. Write a brief narrative of the movement for a shorter working day.
  4. Review the relations between organized labor and the steel industry in the United States.

C

Take from this group at least two and not more than four

  1. What is “the labor problem”?
  2. Compare American and British labor leadership. How do you account for the differences?
  3. “Employers must be free to employ their work people at wages mutually satisfactory, without interference or dictation on the part of individuals or organizations not directly parties to such contracts.” Comment.
  4. Discuss a proposed law providing that “in the establishment of salaries for school teachers in the city of—, there shall be no discrimination based on sex or otherwise, but teachers and principals rendering the same service shall receive equal pay.”
  5. “The principle that each industry shall support its own unemployed is one that must be established if a real solution of unemployment is to be made.” Do you agree? Why, or why not?
  6. Discuss the relation of shop committees to trade-unionism.

_______________________________

DIVISION SPECIAL EXAMINATION
ECONOMICS AND SOCIOLOGY

Answer six questions

A

Take from this group at least one and not more than two

  1. Discuss the following contention: “The landlord is a parasite since he consumes without producing.”
  2. What is the meaning of “over-population”?
  3. “Hitherto it is questionable if all the mechanical inventions yet made have lightened the day’s toil of any human being.” Comment critically.
  4. What are the interactions of human instincts and modern factory labor?
  5. Discuss the nature and bases of economic prosperity.

B

Take from this group at least one and not more than two

  1. Describe the evolution of language.
  2. Trace the history of the middle class and forecast its future.
  3. Give a brief historical account of the status of women.
  4. What have been the chief cultural consequences of the machine process?

C

Take from this group at least two and not more than four

  1. What is the province of sociology?
  2. Discuss the family as a necessary social unit.
  3. Describe the leading forms of conflict and their effect upon group life. Why are some forms to be preferred to others? What are the factors which determine the forms actually prevailing at any time?
  4. Analyze the sources of prestige and influence in modern society.
  5. “From the standpoint of progress, the value of the individual depends on the excess of his production over his consumption.” Discuss.
  6. What are the criteria and causes of racial superiority?

_______________________________

Examinations not transcribed for this post

History:

General Examination
Special Examinations: Mediaeval History; English History; Modern European History to 1789; Modern History since 1789; American History

Government:

General Examination
Special Examinations: American Government; Municipal Government; Political Theory; International Law

_______________________________

Source: Harvard University Archives. Divisional and general examinations, 1915-1975 (HUC 7000.18). Box 6, Bound Volume (stamped “Private Library Arthur H. Cole”) “Divisional Examinations 1916-1927”.

Categories
Exam Questions Queen's University Theory

Queen’s University at Kingston. Comprehensive Economic Theory Exam in Economic Theory, 1974

Your curator of the economic artifacts so lovingly transcribed here for future digital generations of historians of economics, human or artificially intelligent, has a weakness for orphaned examinations that he discovers upon riffling through the archival papers of defunct professors of economics. While I have no illusions about being able to give each worthy artifact I stumble upon a warm post in this collection, perhaps others will be inspired by my efforts to join in. Transcribe and upload!

About a dozen years ago I found a folder of University of Chicago economics exams in George Stigler’s papers in which rested a forlorn ten-page Ph.D. comprehensive examination in economic theory from Queen’s University at Kingston, Ontario, Canada (May 1974). I found no clue to the backstory of how this exam happened to have found its way to George Stigler’s papers. Was it from Stigler demand (“Could you send me a copy of your most recent Ph.D. exam in economic theory”) or was it a Queen’s colleague’s pride of workmanship that resulted in the supply? My maximum-likelihood-gut-instinct estimate is that the source was Richard Lipsey, then Sir Edward Peacock Professor of Economics, but it would come as no surprise if there was some Chicago Ph.D. alum on the faculty then at Queen’s who shared the examination with Stigler out of fealty. But the backstory is unimportant for our purposes here, below you will find an excellent comprehensive exam that expands our international cross-section of economic theory questions. Economics in the Rear-view Mirror hereby raises the following artifact from literal archival obscurity to virtual visibility. 

_______________________

QUEEN’S UNIVERSITY AT KINGSTON
Department of Economics

Ph.D. Comprehensive Examination
ECONOMIC THEORY
May, 1974

Answer FOUR (4) questions — one from each of the four parts of the examination. All questions are of equal value.

PART I

  1. Consider a world in which there are two commodities, beer and whisky and four individuals described by the following utility functions:

Tom: UT = 2B + W
Sally: US = B + W
Dick: UD = min (B, W)
Jane: UJ = min (B/2 , W)

where utility (U) is measured in utils, beer (B) in bottles and whisky (W) in shots. Bottles of beer and shots of whisky are perfectly divisible.

    1. Draw the one-util indifference curve for each of these individuals.
    2. Construct Tom’s and Dick’s uncompensated demand curves for beer under the assumption that each has a money income of $10 and the price of whisky is 50¢ a shot.
    3. How much beer would Tom and Dick each consume if their money incomes were each $10 and a bottle of beer and a shot of whisky cost 25¢ and 50¢ respectively? Construct each of their compensated demand curves for beer through this point.
    4. Suppose that Tom and Sally are stranded on an island with 15 shots of whisky and 10 bottles of beer. Use an Edgeworth box diagram to derive the set of efficient (Pareto optimal) allocations of beer and whisky between them.
    5. Suppose that Tom had been stranded with Jane rather than Sally in Part (d). Derive the set of efficient allocations of the 15 shots of whisky and 10 bottles of beer between Tom and Jane.
    6. Suppose finally that Dick and Jane had been caught in the predicament described in Part (d). Derive the set of efficient allocations of the beer and whisky in this case.
    7. Suppose that Dick and Jane had decided to use a price system to sustain one of the efficient allocations in Part (f). What would be the effect on the price of whisky of a decrease in the endowment of whisky from 15 to 12 shots?
  1. Assume a simple Keynesian type model in equilibrium with unemployed resources. Consider the effects on the velocity of circulation of money of (a) an upward shift in the consumption function, and (b) an open market purchase by the central bank. Answer the question under each of the following alternative assumptions.
      1. The money supply is fully exogenous.
      2. The money supply is partly exogenous and partly endogenous, the endogenous part being a function of national income and “the” rate of interest.
      3. The money supply is wholly endogenous because the monetary authorities are stabilizing domestic interest rates.

In answering this question assume a general demand for money function of the form

MD = L(Y,r)

in which the only restrictions are LY > 0, Lr  ≤ 0.

You should also pay attention to the following two special cases:

a) Lr = 0, LY > 0 and b) Lr > 0, (Y/M)LY = 1.

In so far as you have time, give explanations of your results.

  1. Make the following assumptions about the market for cloth and yarn in a small country:
    1. One spool of yarn is required for the manufacture of one bolt of cloth.
    2. The world price of yarn is $1 per spool and the world price of cloth is $3 per bolt.
    3. The domestic demand curve for cloth, the domestic supply curve of yarn, and the domestic supply curve of factors of production (other than yarn) in the manufacture of cloth have constant elasticities, and these elasticities are -1, 1 and 2 respectively.
    4. In free trade, domestic consumption of cloth is 1,000,000 bolts, domestic production of yarn is 100,000 spools, and domestic production of cloth is 200,000 bolts.
      1. What tariff on cloth would just compensate factors of production in the manufacture of cloth for the imposition of a 15 percent tariff on yarn?
      2. What is the total welfare loss of imposing both tariffs?
      3. What would be your answer to (ii) above if the 15 percent tariff on yarn were imposed without the compensating tariff on cloth?
      4. In general, if there is to be a tariff on yarn, how does one determine the welfare implications of imposing a tariff on cloth, too?
      5. How would your answers to Parts (i) and (ii) above be affected if the elasticity of substitution between yarn and other factors in the manufacture of cloth were greater than zero?

PART II

  1. Consider an economy producing two goods, X1 and X2, according to constant returns to scale production functions of the following form:

(1)        X1 = F1(L1)
(2)        X2 = F2(L2, X12)

where Xi is the gross output of the i-th good, X12 is the amount of X1 used as an input in the production of X2, Li is the amount of labour used in the production of Xi and the total supply of labour is fixed

{L_{1}+L_{2}=\bar{L}}

Tastes in this economy can be represented by the utility function

U = AXi.25 X2.75

A) Suppose that equations (1) and (2) are of the particular form:

(1′)       X1 = 2L1
(2′)       X2 = L2

and the total labour supply is 200 units.

    1. Construct the economy’s production possibilities set.
    2. What will be the effect on the equilibrium relative price of X1 of:
      1. an increase of the labour force to 225 units,
      2. a doubling of the efficiency of labour in the X1 sector?

B) Suppose now that equations (1) and (2) are of the particular form:

(1”)      X1 = 2L1
(2”)      X2 = min (L2, 4X12)

Answer Questions A(a) and A(b) under these circumstances.

C) Answer the same questions in the case where a second production process is made available to the X2 sector so that the production functions are of the form:

(1”’)     X1 = 2L1
(2”’)      X2 = (min (L2, 4X12)   or   min (.75L2, 8X12)

D) What would be the general shape of the production possibility curve if X2 could be produced according to a differentiable concave constant returns to scale production function of the general form of (2) (e.g., Cobb-Douglas) while the production function for X1 remained X1 = 2L1?

  1. Standard demand theory predicts that if a man buys more apples when his income goes up he buys less apples when their price goes up. Say whether this prediction continues to follow, and why or why not, under each of the following circumstances.
      1. The quality of apples is judged by their price.
      2. The man is upset by his neighbour’s discarded apple cores.
      3. The man owns an apple orchard.
      4. The man loves the scent of apple blossoms.
      5. Apples are storable cheaply enough that one can speculate in them.
      6. The man drinks cider, the market price of which is positively related to the price of apples.
      7. Apples are a heavily advertised good.
      8. Apples and flour are only consumed in fixed proportions, in the form of apple pies.
      9. The price of apples divided by the price of pears is not equal to the man’s marginal rate of substitution between apples and pears.
      10. All goods are rationed, in such a way that they must be paid for not only with money but also with ration coupons, each consumer having a fixed total coupon allowance. (Coupons may not be bought and sold.)

6. [Capital theory and functional income distribution]

    1. You are dealing with an economy in which there is only one product which is used either as a consumption good or as a capital good. Each period’s output is divided in some proportion between consumption use and additions to the capital stock. Capital goods do not depreciate. There are two factors of production, labour and capital goods, available in fixed supply in the short period. There is no unemployment of any factor that has a positive price.
      1. In what circumstances will the functional distribution of income be determined by conditions in the factor market only?
      2. Given that investment is exogenously determined and consumption is functionally related to incomes, in what circumstances might the functional distribution of income be determined from conditions in the product market?
      3. In what circumstances would the distribution of income be indeterminate?
      4. In what circumstances might conditions in the factor and product markets be inconsistent with a determinate distribution of income?
    2. Now consider an economy with two consumer goods, one of which can also be used as the single capital good, both products being produced by labour and capital goods. Again there can be positive investment in each period and there is no depreciation.
      What would your answers to questions (i), (ii), (iii), and (iv) of part (a) be now?

PART III

  1. An economy producing only one commodity, which may be used either as a consumer good or as a capital good, has a production function of the form

{Y_{t}=e^{\lambda t}K^{\alpha }_{t}L^{1-\alpha }_{t}}

and we are given that

{L_{t}=e^{gt}L_{0}}

where Yt is the quantity of output; Kt  is the capital stock; Lt the number of units of labour; g is the rate of growth of the labour force; {\lambda}  is the rate of Hicks neutral technological change; and {\alpha} takes the values 0 < {\alpha} < 1.

a) [“Golden-rule”]

      1. Derive the general formula that determines the golden rule capital-labour ratio when there is no technical change.
      2. Let g = 0.02, {\lambda} = 0, {\alpha} = 0.25. Calculate the golden rule level of the capital-labour ratio. (Carry your calculation to the last stage in which the solution is set up in the form of logs or as a power of a number.)
      3. Now let the growth rate of the labour force rise to .03. Find the ratio of the new golden rule capital-labour ratio, k1, to the old golden rule capital-labour ratio, k0.
      4. Maintain the growth rate of the labour force, g1 at .03 and make {\lambda} = .0225. Obtain the golden rule values of the path of the capital-labour ratio, labour being measured in natural units.

b) [Optimal steady state growth]

      1. Derive the rule that determines the capital-labour ratio of the optimal steady state growth path when there is positive subjective discounting of future per capita utilities at the rate, {\rho}.
      2. Assume that g = .02, {\lambda} = 0, {\alpha} = 0.25 and {\rho} = .04. Calculate the capital-labour ratio of the optimal steady state growth path.

8. [Keynesian macroeconomic theory]

    1. Investigate the consequences for fiscal and monetary policy in a simple Keynesian model of assuming that consumption expenditure is unrelated to current income (because it is related to permanent income which is assumed constant over the time period in which we are interested). Would it matter whether or not there were some other element of aggregate expenditure that was related to current national income?
    2. Explain how the permanent income hypothesis reconciles the observations drawn from long and short period time series and cross section studies that appeared inconsistent with the Keynesian consumption function. Is the weight of evidence such that we can have substantial confidence in the truth of the general concept that consumption relates to permanent rather than current income?
    3. Briefly discuss the significance for the efficacy of short-term fiscal policy of establishing that consumption is a function of permanent income rather than current income.
  1. Examine the validity of the following statements: a) when wages are only flexible upwards and b) when wages are fully flexible.
      1. Government spending in excess of tax revenue is inflationary in the long run.
      2. Union wage pressure can reduce unemployment due to the stimulating effects of higher wages on aggregate demand.
      3. Sooner or later automation will cause unemployment.
  1. “Thinking about saving and investment from this technocratic point of view has convinced me that the central concept in capital theory should be the rate of return on investment. In short we really want a theory of interest rates, not a theory of capital.” (Solow: Capital Theory and the Rate of Return, p. 16. Italics in the original.)
    1. What is the relationship between the two sentences in Solow’s quotation that makes the second sentence follow from the first? In your answer elaborate on the concept of the rate of return.
    2. Can stocks of capital goods (or the aggregate of asset values) be more or less excluded from capital theory, which among other things must include a theory of intertemporal choice?

PART IV

  1. Various officials and ministers of the Canadian government have expressed concern and implemented or at least suggested policies in response to recent increases in the world prices of certain commodities. One instance has been the imposition of a tax on exports of Canadian oil. This move has been hailed by members of the government on various grounds: its “anti-inflationary consequences”, its “benefit to the poor and to fixed income classes to whom cheap energy is so vital” and “the stimulus it will provide to the Canadian manufacturing sector”. Some observers have been obviously impressed with the success of this policy and have suggested a similar two-price system for certain Canadian agricultural exports because “there is no reason the Canadian consumer should suffer due to a world scarcity of these commodities when there is an abundant supply in Canada.”
    1. Attempt to give a precise formulation to the exact policy goals which might be implied by each of the above quotations.
    2. How would an export tax help to achieve each of these goals?
    3. What would be the nature of the economic costs resulting from the use of an export tax to achieve these goals and what would determine the magnitude of these costs?
  1. A certain country, previously without an unemployment insurance system, sets one up. Its rules are as follows:
      1. A minimum of 10 weeks work must be done before any benefits can be drawn.
      2. For each week worked above 10, a person is entitled to two-thirds of a week on unemployment benefits, except that no more than 12 weeks of benefit can be received in a year.
      3. While on benefit the weekly payment is three-quarters of the weekly wage previously earned.

To simplify the problem, the following assumptions may be made: how the system is financed is irrelevant; each year is self-contained, in the sense that benefits earned must be used up in the year or lost; one can draw benefit even if one quits voluntarily; no time is ever lost in finding a job if one wants one; a year is 50 weeks long.

    1. Establish that a person’s income-leisure opportunity locus consists of four line segments, and draw them on the accompanying graph paper.
    2. Will anyone ever work exactly 10 weeks under the new unemployment insurance system? Explain.
    3. If leisure is a superior good, prove that those who used to work between 28 and 38 weeks a year (before the new unemployment insurance system) will now work less.
    4. Define a “full-time worker” as one who works more than 38 weeks a year. Would you expect such a worker to work less after the scheme is introduced?
    5. Demonstrate the possibility that people who did not work before will now work. Could they conceivably become “full-time” workers? Why or why not?
  1. Consider a country with the following attributes:
      1. It has a floating exchange rate.
      2. It has a substantial rate of inflation which it is trying to lower.
      3. It has recently taken monetary action that has raised interest rates relative to those of other countries.
      4. It is buying quite large amounts of foreign exchange, apparently to have a dampening effect on the rise in the exchange value of its currency.
      5. This foreign exchange is acquired by use of government cash which requires extra government borrowing to the extent that the exchange accumulation was not foreseen (the country has been lucky in its foreign exchange receipts and disbursements for oil roughly matching).
      6. The country has outstanding large amounts of non-negotiable savings bonds which are redeemable on demand at face value, which have coupon rates considerably lower than rates of comparable securities, and which are being cashed in substantial quantities.
    1. Are the objectives implied by the above actions mutually consistent? Always? In some circumstances? Never? In your answer elaborate on the relevance of some of the parameters of the system to it.
    2. The federal government of this country will be bringing down its annual budget soon. What policies would you recommend (i) to alleviate inconsistencies in the implied objectives noted above, and (ii) to reinforce the impact of the actions described above?

Source: University of Chicago Archives. George Stigler Papers, Addenda. Box 33. Folder “Exams & Prelim Questions.”

Image Source: “Coat of Arms” in the on-line Queen’s Encyclopedia.

Categories
Exam Questions Johns Hopkins Theory

Johns Hopkins. Exam for Welfare Economics. Lerner, 1958

 

Abba Lerner changed his academic locations (including leaves of absence to accept visiting positions) with a frequency rivaled by few. The academic year 1957-58 found him visiting the department of political economy at Johns Hopkins University. The artifact for this post is the final examination for Lerner’s course on welfare economics.

Lerner’s notes for seminars on social welfare functions held at the IMF and at the Cowles Commission in  1952.

_________________________

THE JOHNS HOPKINS UNIVERSITY
Welfare Economics
18-640

Final Examination May 20th 2-5 p.m., 1958.
Abba P. Lerner

Answer four questions, in separate blue books, in ink.

  1. Discuss the meaning, the validity and the significance of the proposition that it is impossible to derive a social welfare function from individual preference functions.
  2. How far can one carry the analogy between a political voting procedure and the economic price mechanism, and between the rationale of voting between alternative policies and that of allocating dollars between alternative purchases?
  3. Discuss the rational elements in relation to other elements in the social objectives of optimum distribution of income, optimum population, and optimum rate of saving.
  4. What is sound, what is unsound, and what is useful in the doctrine of consumers’ surplus?
  5. Why is it socially desirable to have the prices of products equal to the value of the marginal factors used in their production? How is this objective affected by equity elements such as the need for subsidies?
  6. Under what conditions would a partial freeing of trade be harmful to society in the largest sense? In your answer explain the treatment of this problem in terms of “second best” and the use of the concept of “divergence”.
  7. Compare the arguments for the imposition of trade restrictions for the sake of affecting the international terms of trade with those undertaken for the sake of affecting the domestic distribution of income. Give special attention to the interdependence of efficiency and equity considerations.

Source: Johns Hopkins University. The Eisenhower Library. Ferdinand Hamburger, Jr. Archives. Department of Political Economy [Records], Series 6/7, Box 3, Folder “Department of Political Economy, Graduate Exams 1933-1965”.

Image Source: Photograph of Abba Lerner printed in an announcement for his speech “Israel—The Next Ten Years” (February 25) at the 1958 Forum presented by Beth Emet the Free Synagogue (Evanston, Illinois). Library of Congress, Manuscript Division. The Papers of Abba P. Lerner, Box 6, Folder 8 “ ‘B’ miscellany”. A copy of the announcement was posted by Ellen Blum Barish in Tablet (January 14, 2014).

Categories
Exam Questions Suggested Reading Syllabus Theory

Queens College. Reading assignments and exams for macroeconomics. Lerner 1973-1975

Economics has its share of Wunderkinder, “Primo Donnos”, and heterodoxic poseurs. It is also a fact that economists are overwhelmingly herd animals. From time to time we find a genuine maverick among us, Abba Ptachya Lerner could be designated the poster-child of maverick economists. 

In this post Economics in the Rear-view Mirror has assembled material over three consecutive years from his seminar in advanced macroeconomic theory offered at Queens College of the City University of New York from the 1970s. Class schedules, assigned readings, midterm and final exams are transcribed here from the Spring terms of 1973 through 1975.

_______________________________________

Course Description (1974)

Economics 710, 80:
Seminar in Advanced Macroeconomic Theory
Abba Lerner

An integration of the theories of employment, inflation, interest, capital, investment, and growth, and new lessons for the uses of monetary policy, fiscal policy, and price policy. The Keynesian revolution (interpretations and misinterpretations—general theory or special case?), pre-Keynesian, Keynesian, and post-Keynesian economics. International complications and the myth of international money.

Basic Reading
Ackley Macroeconomics
Keynes The General Theory of Employment, Interest and Money
Leijonhufvud On Keynesian Economics and the Economics of Keynes
Lerner “Money” (Encycl. Britt., 1946 ed.)
Everybody’s Business
Flation
Other Suggested Readings
Lekachman Keynes’ General Theory – Reports of Three Decades
Harrod Life of Keynes
Lerner The Economics of Employment

There will be one midterm test and a final Examination.

Source: Queens College of the City University of New York. Economics, Spring 1974 (Economicsdepartment brochure), p. 20. Copy in Papers of Abba P. Lerner, Box 17, Folder 6 “Queens College of the City University of New York: General”, U. S. Library of Congress, Manuscript Division.

_______________________________________

1973

QUEENS COLLEGE
DEPARTMENT OF ECONOMICS

Economics 710, 80: Advanced Macroeconomic Theory
Inflation, Employment and Growth
Seminar, Spring 1973

Prof. Abba Lerner
Th. 6:30-8:20 P.M.
SS 314

Reading
Branson Macroeconomics Theory and Policy
Harper & Row
(B)
Lerner Flation
Quadrangle Books
(F)
Lerner Everybody’s Business
Harper Torchbooks (paperback)
(EB)
Lerner Money
(Encycl. Britt 1946 edition)
(M)
Leijonhufvud On Keynesian Ecs & the Ecs of Keynes
Oxford U P
(L)
Keynes The General Theory of Interest and Money (sic)
Harcourt Brace
(K)
Tentative Outline
Week Date
1 Feb. 8 Introduction B:1-3 / M / EB:X.
2 15 The Classical Case B:4-6 / F:1-5 / K.
3 22 Static Equilibrium B:7-9 / F:6-7 / K.
4 Mar 1 Consumption and Investment B:10,11 / K.
5 8 Money B:12,13 / M / K.
6 15 Monetary & Fiscal Policy B:14 / K.
7 22 The Foreign Sector B:15 / F:16,17 / K.
8 29 International Money F:18-20 / K.
9 Apr 5 Inflation B:16 / EB:XI / F:8-15 / K.
10 12 Unemployment Disequilibrium L: I and II / K.
11 19 Macromodels L: III / K.
12 26 Liquidity Preferences L:IV / K.
13 May 3 Keynes and Post Keynes L:V and VI / I(sic).
14 10 Growth Models B:17-19.
15 17 Optimum Growth Models B:20-23.
Other Suggested Readings
Lerner Economics of Employment
Ackley Macroeconomics
Lekachman Keynes’ General Theory – Reports of Three Decades
Lekachman The Age of Keynes
Harrod Life of Keynes

Source: U. S. Library of Congress, Manuscript Division. Papers of Abba P. Lerner, Box 17, Folder 4 “Queens College of the City University of New York: Course outlines. 1971-77, n.d.”.

*  *  *  *  *  *  *  *  *  *  *  *  *  *

QUEENS COLLEGE
DEPARTMENT OF ECONOMICS

Economics 710 and 80
Prof. A. Lerner
Midterm Examination
March 22, 1973

Answer two questions from each Part

PART ONE

  1. M = 300, V = 4, C(Y) = 5/6, I = 300 – 10i
    (I = Investment), (1 = rate of interest)
    What would be the equilibrium values of Y, i, I, and S?
    What would happen to those if

    1. there was an increase in liquidity preference?
    2. M was increased to 450?
    3. C(Y) increased to 7/8?
  1. What is the multiplier? Now is it similar to and how different from the velocity of circulation of money? the accelerator? the balanced budget multiplier?
    How would it be affected by

    1. a change in liquidity preference?
    2. a change in time preference
    3. a change in the elasticity of supply of money?
    4. a change in the propensity to consume?
    5. a fixed M and v?
  1. Describe carefully the mechanism by which an increase in M would increase S. How would this be affected if a lower i decreased the amount people wanted to save?

PART TWO

  1. Discuss these statements:
    “The main contribution of Keynes was to point out that full employment is not reached automatically because

    1. cannot become negative.
    2. workers have no way of reducing their real
    3. workers insist on a real wage greater than their marginal product.
    4. There is a liquidity trap.
  2. Describe carefully the meaning of the marginal efficiency of capital and the marginal efficiency of investment, and how they are related.
    1. in a growing economy
    2. in a declining economy
    3. in a stationary economy
    4. in a steady state of growth economy.
  3. “The rise and fall of the Phillips curve.”
    or
    “ The natural level of unemployment.”

Source: U. S. Library of Congress, Manuscript Division. Papers of Abba P. Lerner, Box 17, Folder 5 “Queens College of the City University of New York: Examinations”.

*  *  *  *  *  *  *  *  *  *  *  *  *  *

QUEENS COLLEGE
DEPARTMENT OF ECONOMICS

Economics 710 and 80
Mr. Lerner
Final Examination
Spring 1973

Answer Question 1 and three others.

  1. Branson Page 24 “the saving investment identity…as a natural derivative from the GNP identity”
    Branson Page 26 “The basic GNP identity:—

C + I + G + (X -M) = GNP = C + S + T + Rf,”

[where]

C=616; I=135; G=219; (X-M)=4; GNP = 974;
S = 149; T=208; Rf=1

    1. Show how the S, I identity can be derived from the GNP identity, spelling out any definitional differences in S or I.
    2. Give (and explain) another definition of S or I, as used by economists that makes I necessarily equal to S only in equilibrium.
  1. Compare the “classical” with the “keynesian” explanations of the automatic establishment of full employment on the assumption of wage and price flexibility.
  1. Explain how an increase in thrift (the desire to save) could (or could not) have the effect of (a) increasing investment, (b) decreasing investment, (c) increasing total income, (d) decreasing total income.
  1. Permanent Income, Life-cycle Consumption Hypothesis, Time Preference, Ratchet Effect, Wealth Effect, Pigou Effect — How are these six items related to the consumption function and to each other?
  1. Explain Branson’s distinction between the “real wage model” and the “money wage model”, and the purpose of the distinction.
  1. What do you consider the most distinctive feature of Leijonhufvud’s approach?
  1. Discuss.
    1. “With perfect wage and price flexibility there can be no problem of involuntary unemployment.”
    2. “A little wage and price flexibility (such as might be achieved in practice) could be worse than none.”
    3. “Too much wage and price flexibility could also cause trouble and not provide stable full employment.”
  1. Why is a rise in the price of foreign currency considered more of a crisis than a rise in the price of coffee or Mutual Fund Shares or real estate?

Source: U. S. Library of Congress, Manuscript Division. Papers of Abba P. Lerner, Box 17, Folder 5 “Queens College of the City University of New York: Examinations”.

_______________________________________

1974

QUEENS COLLEGE
DEPARTMENT OF ECONOMICS

Economics 710, 80
Inflation, Employment and Growth
Seminar in Advanced Macroeconomic Theory
[1974]

Abba Lerner
Tues. 6:20-8:00 P.M.
Temp. 3, Room 2

An Integration of the theories of Employment, Inflation, Interest, Capital, Investment and Growth, and its lessons for the uses of Monetary Policy, Fiscal Policy and Price Policy. The Keynesian Revolution (Interpretations and Misinterpretations – General Theory or Special Case?) Pre-Keynesian, Keynesian and Post Keynesian Economics. International Complications and the Myth of International Money.

Basic Reading
Ackley Macroeconomics A
Breit & Ransom The Academic Scribblers B
Keynes The General Theory of Employment, etc. K
Leijonhufvud On Keynesian Economics, etc. L
Lerner Money (Encycl. Britt 1946 edition) M
Lerner Everybody’s Business EB
Lerner Flation F
Other Suggested Readings
Lekachman Keynes’ General Theory – Reports of Three Decades
Harrod Life of Keynes
Lerner The Economics of Employment

There will be one midterm test and a final Examination.

Week Date
1 Feb. 5 Outline — Classical to Keynes M, A 1-4, EB 10, F 1-5
2 12 Lincoln’s Birthday — no classes
3 19 Outline — Post Keynes A 5-8, EB 11, F 6-10
4 26 Say’s Law, Saving and Investment EB 13-14, F11-15
5 Mar 5 Monetary Policy A 9
6 12 Fiscal Policy, Consumption Function A10-12
7 19 The Complete Keynes Model A13-15
8 26 Inflation and Investment A16-17
9 Apr 2 Capital and Growth A18-19
10 9 No Class (Recess)
11 16 International Money F16-20
12 23 Keynesian Revolution? Dynamics L Parts I, II
13 30 Macromodels L Part III
14 May 7 Liquidity and Wealth L Part IV
15 14 Expectations, Illusions and Policies L Parts V, VI
16 21 Review M, A 20, EB, F

Source: U. S. Library of Congress, Manuscript Division. Papers of Abba P. Lerner, Box 17, Folder 4 “Queens College of the City University of New York: Course outlines. 1971-77, n.d.”.

*  *  *  *  *  *  *  *  *  *  *  *  *  *

QUEENS COLLEGE
DEPARTMENT OF ECONOMICS

Econ. 80—710
MIDTERM
Mr. Lerner
April 16, 1974

Answer question 1 and 3 other — 4 altogether.

  1. Suppose there is a decrease in the propensity to consume. Show how would this affect Employment, Prices and Investment
    1. if the amount of money is held constant
    2. if the rate of interest is held constant
      with wages and prices (1) perfectly flexible; (2) sticky downward
  1. Explain the relationship between the marginal efficiency of capital and the marginal efficiency of investment.
    How are these affected by an increase in
    1. the rate of interest?
    2. the optimism of investors?
    3. the wealth of the economy?
    4. the rate of time preference?
  1. I-S, L-M. Explain the nature of these curves, their use, and the stability condition they demonstrate.
  1. Show how the multiplier would be affected by an increase in
    1. the propensity to consume
    2. the marginal propensity to consume
    3. the marginal propensity to save
    4. the quantity of money
    5. the velocity of circulation
    6. liquidity preference
    7. government spending
    8. tax collection
    9. (7) and (8) together
  1. Why must saving (S) always be equal to investment (I)? What could be meant by the statement that an excess of S over I is deflationary?
  1. What are the features which distinguish the Keynesian from the pre-Keynesian and from the Post-Keynesian theories of the equilibrium level of employment? 

Source: U. S. Library of Congress, Manuscript Division. Papers of Abba P. Lerner, Box 17, Folder 5 “Queens College of the City University of New York: Examinations”.

*  *  *  *  *  *  *  *  *  *  *  *  *  *

QUEENS COLLEGE
DEPARTMENT OF ECONOMICS

Economics 80, 710
Inflation, Employment and Growth
Seminar in Advanced Macroeconomic Theory
Dr. A. Lerner

Final Examination
Spring 1974

Answer Question 1 and any three other questions — Four altogether

  1. Suppose there is an increase in thriftiness. Show the direct and the indirect effects on the Multiplier, Income, Employment, Prices and Investment
    1. with wages and prices perfectly flexible,
    2. with wages and prices sticky downward,
      in each case

      1. with the quantity of money held constant
      2. with the rate of interest held constant
  2. State and discuss the primary criticisms that Leijonhufvud makes of a) Keynes and b) the “Keynesians”.
  1. Compare the advantages and disadvantages of indexing
    1. when there is a steady rate of inflation and
    2. when there is a danger of inflation getting out of control.
  1. A well informed and intelligent observer remarks that “We are now suffering from an excess of saving over investment, which is deflationary”. He cannot really mean what he is saying because in the first place we are suffering from inflation rather than deflation and in the second place it is not possible for saving to be greater (or less) than investment. What could he be meaning to say?
  1. Marginal productivity of capital; Marginal productivity of investment; Marginal efficiency of capital; Marginal efficiency of investment.
    Under what conditions are any of these equal to the rate of interest? Explain carefully.
  1. How come America allowed the dollar to depreciate on the international money market? How is this different from national bankruptcy? Will this not result in the disorganization of international trade “beggar thy neighbor” competitive devaluations of freely floating currencies. How could it have been prevented and why was it not prevented?
  1. “The present very high interest rates show the determination of the government to stop the inflation. If this policy is persisted in it is bound to achieve this purpose since, as we all know, the fundamental cause of inflation is the increase in the quantity of money which the authorities have provided in the recent past in order to keep down interest rates”. Discuss the logic and the practicality of such a policy.
  1. How is the size of the multiplier likely to be affected by an increase in:
    1. the rate of interest?
    2. time preference?
    3. liquidity preference?
    4. government spending?
    5. the quantity of money?
    6. the population?
    7. the government budget?
    8. the use of charge accounts for consumer credit?

Source: U. S. Library of Congress, Manuscript Division. Papers of Abba P. Lerner, Box 17, Folder 5 “Queens College of the City University of New York: Examinations”.

_______________________________________

1975

QUEENS COLLEGE
DEPARTMENT OF ECONOMICS

Economics 710, 80
Inflation, Employment and Growth
Seminar in Advanced Macroeconomic Theory
Abba Lerner
Spring 1975
Temp. 3, Room 2

An Integration of the theories of Employment, Inflation, Interest, Capital, Investment and Growth, and its lessons for the uses of Monetary Policy, Fiscal Policy and Price Policy. The Keynesian Revolution (Interpretations and Misinterpretations – General Theory or Special Case?) Pre-Keynesian, Keynesian and Post Keynesian Economics. International Complications and the Myth of International Money.

Basic Reading
Ackley Macroeconomics A
Keynes The General Theory of Employment, etc. K
Lerner Money (Encycl. Britt 1946 edition) M
Lerner Everybody’s Business EB
Lerner Flation F
Other Suggested Readings
Lekachman Keynes’ General Theory – Reports of Three Decades
Lerner The Economics of Employment

There will be one midterm test and a final Examination.

Week Date
1 Feb. 6 Outline — Classical to Keynes M, A 1-4, EB 10, F 1-5
2 13 Outline — Post Keynes A 5-8, EB 11, F 6-10
3 20 Say’s Law, Saving and Investment EB 13-14
4 27 Monetary Policy A 9
5 Mar 6 Fiscal Policy, Consumption Function A10-12
6 13 The Complete Keynes Model A13-15
7 20 No Class (Recess)
8 27 Midterm test
9 Apr 3 Inflationary Depression, the Wage Unit A16
10 10 Capital and Growth A17-18
11 17 International Money F16-20
12 24 Keynesian Revolution? Dynamics Lerner JEL Mar 1974
13 May 1 Liquidity and Wealth A 19
14 8 Expectations, Illusions and Policies A 20, F 11-15
15 15 Review

Source: U. S. Library of Congress, Manuscript Division. Papers of Abba P. Lerner, Box 17, Folder 4 “Queens College of the City University of New York: Course outlines. 1971-77, n.d.”.

*  *  *  *  *  *  *  *  *  *  *  *  *  *

Economics 710-80
Midterm Exam

Abba P. Lerner
March 20, 1975

Answer Question 1 and any two other questions (three altogether)

  1. If the propensity to consume (average and marginal) increases from 50% to 60% what would be the effect on the level of income?
    In your answer consider the six (6) possible combinations of the following conditions:
    The mpI (marginal propensity to invest, i.e. the increase in investment due to an increase in income as a percentage of the increase in income) is (a) 40% (b) 30%
    The elasticity of supply of money plus the elasticity of demand for money is (i) infinite (ii) unitary (iii) zero
  1. Discuss your views and those of Keynes, Ackley, and Lerner on “Involuntary unemployment is basically due to the inability of workers to reduce their real wage”.
  1. Which (if any) of [the following] statements are true? Why do you think so? Indicate any connections between them.
    1. “The more steeply any average (A) is rising (falling) the more will the corresponding marginal (M) be above (below) it, i.e. the greater will be M minus A”.
    2. “The short run mpC (marginal propensity to consume) is less than the short run apC (average propensity to consume)”.
    3. “The long run mpC is equal to the long run apC”.
    4. “For a temporary increase in income mpC minus apC is less than for a permanent increase in income”.
  1. What is Say’s Law? Discuss its logical base, its empirical validity and its practical usefulness.
  1. Income can be defined more widely or more narrowly. Distinguish between the different definitions and indicate how the different definitions could better serve different purposes.

Source: U. S. Library of Congress, Manuscript Division. Papers of Abba P. Lerner, Box 17, Folder 5 “Queens College of the City University of New York: Examinations”.

*  *  *  *  *  *  *  *  *  *  *  *  *  *

QUEENS COLLEGE
DEPARTMENT OF ECONOMICS

Economics 710/80
Dr. A. Lerner
FINAL EXAMINATION
May 22, 1975 (Thursday)

Temp 3 Room 2

Answer Questions 1 and any two others — 3 altogether

  1. Suppose consumption increases from 70% of GNP to 80% and the marginal propensity to consume increases from 50% to 75%. What could be the effect on the GNP if the elasticity of demand for money plus the elasticity of supply of money (with respect to changes in the rate of interest) is (a) infinite (b) zero (c) one, and the marginal propensity to invest (with respect to GNP) is (and remains) (i) 20% (ii) 25% (This makes six combinations).
  1. Is an increase in the national debt beneficial, harmful or neutral for the welfare of (a) the present generation (b) future generations? State and examine the arguments for the different views.
  1. “Involuntary unemployment is due to the inability of workers to reduce their real wage”. “Involuntary unemployment is due to an unsatisfied demand for a larger stock of money”. Discuss.
  1. “To succeed in winning the battle against stagflation we must (a) tighten our belts (b) use the tax rebate to buy more automobiles and (c) make more money available for mortgages for housing”. Discuss.
  1. “Say’s Law is invalid but useful while the Pigou Effect is valid but useless.” Does this make any sense?
  1. Does the rate of interest determine the marginal efficiency of capital? Or Vice versa? Or what?

Source: U. S. Library of Congress, Manuscript Division. Papers of Abba P. Lerner, Box 17, Folder 5 “Queens College of the City University of New York: Examinations”.

Image Source: National Academy of Sciences. 1994. Biographical Memoirs: Volume 64, p.208.  Washington, DC: The National Academies Press.

 

Categories
Chicago Exam Questions Theory

Chicago. First quarter price theory exams. Rees, 1960

Happy to add another round of first quarter price theory exams from the University of Chicago to the collection. Always nice to have a picture from the early professional years of the economists featured here. Distinguished old farts were once rising stars after all. (A general wisecrack made with the qualification, “present company excluded”.)

______________________

Posted earlier

Reading list and exams from the Autumn quarter of 1962.

______________________

Economics 300
Mr. Albert Rees

Midterm Examination
Autumn 1960

  1. (16 points) State whether each of the following statements about the U. S. economy is true, false, or uncertain, and explain your answers briefly.
    1. Consumers decide what will be produced.
    2. All consumers participate equally in determining what will be produced.
    3. The government influences the composition of output in the private consumer goods sector.
    4. The government determines the level of investment for the economy as a whole.
  1. (10 points) Comment briefly on the following statement:
    “When equilibrium prices in competitive markets are disturbed, they tend to be re-established. Thus the first effect of an increased supply of eggs is to lower the price. At this lower price, consumption is increased, and the increase in demand tends to drive the price back up again.”
  2. (16 points) Increased costs cause manufacturers to reduce the size of 5 cent chocolate bars from 2-1/2 ounces to 2 ounces. Because the bars are smaller, people eat more of them and consumption rises from 10, 000 bars a week to 11,000.
    1. Can these events be shown on an ordinary supply and demand diagram? If so, show them. If not, explain why.
    2. Can the elasticity of demand for chocolate be computed? If so, compute it. If not, explain.
  1. (24 Points) The following table gives hypothetical prices of pork and beef per pound in two years, and quantities consumed in a certain town.

Price per pound Pounds consumed
1959 1960 1959

1960

Pork

40 cents 50 cents 1000 800
Beef 60 cents 60 cents 1000

1200

    1. Compute the elasticity of demand for pork and the cross-elasticity of demand for beef in terms of the price of pork.
    2. Compute the Laspeyres price index for the price of meat from 1959 to 1960 (assuming that pork and beef are the only kinds of meat).
    3. Draw an indifference map for pork and beef for a typical consumer and illustrate the changes shown in the table on his indifference map. Derive two points on his demand curve for pork.
    4. Assume that the consumer’s money income is increased by an amount equal to his original income times the Laspeyres price index computed in (b). Demonstrate that he has been overcompensated for the price rise. Under what condition if any would this increase in income fail to overcompensate him?
  1. (16 points) Jones lives in a rented house for which he pays $150 a month. He has the opportunity of buying an identical house for $25,000, of which $15,000 will be paid in cash and $10,000 can be borrowed on a mortgage. He has figured that his monthly expenses would be $100 if he bought: $50 for interest on the mortgage, $20 for local taxes, and $30 for maintenance and depreciation. His income tax and expenses for fuel and utilities will not be affected by the purchase. He argues that it will cost him less to live if he buy the house; his wife argues that it will not.
    1. Under what conditions is Jones right? Under what conditions is Mrs. Jones right?
    2. Is there any divergence between the “right answer” to this problem from the private standpoint of the Jones family and from the standpoint of society? Explain.
  1. (18 points) The GJS corporation, manufacturers of gadgets, have determined that for every 10 per cent increase in the capacity of a gadget factory, minimum short-run average total cost falls by 1 per cent throughout the relevant range of capacities.
    1. What can you say about the production function for gadgets over the relevant range?
    2. Suppose that the company hires two factors of production, labor and capital, and pays each its marginal product. Will anything be left over for the owners of the company who contribute no services? Explain.
    3. Suppose that the company wants to build a plant to produce 10,000 gadget per week. What can you say about the size of the plant that will produce these most efficiently?

Economics 300
Mr. Rees
Fall. 1960

Final Examination
December 14, 1960

  1. (21 points) Show each of the following events on an indifference map:
    1. The change in the consumption of margarine following an increase in income (axes: butter and margarine. Assume that the income elasticity of demand is positive for butter and negative for margarine.)
    2. The change in the consumption of bread following a rise in its price. (axes: bread and all other commodities.) Identify the income and substitution effects of the price change.
    3. Do part (b) over using Friedman’s “Marshallian demand curve” concept and explain the difference between the diagrams for (b) and (c).
  2. (19 points) In the United States, about one-fifth to one-fourth of all income is property income. State briefly (a) the advantages of having private income from property in our economy (b) the costs or disadvantages. You may judge these according to any values you care to use, making the values as explicit as possible.
  3. (20 points) In a certain isolated area there are 50 farms of each of two types, A farms and B farms (100 farms in all). Within each type, all farms are identical. All farms are worked by identical workers. The marginal product schedules of one farm of each type are given below, in bushels of wheat per year.

A Farm

B Farm

No. of workers

1 100

95

2

90 84
3 80

73

4

70 62
5 60

51

  1. If there are 260 workers in the area, how many will be employed on each kind of farm? What is the total product of each kind of farm? The rent of each kind of farm? The wages of workers on each type of farm in bushels per year? (Assume that farmers compete freely for labor, and labor can move within the area.)
  2. By means of an irrigation project, the owners of twenty B farms transform them into A farms. Recompute the answers to (a), counting the transformed farms as A farms. Who gained and who lost from the project, and why?
    1. (20 points) By means of appropriate diagrams and/or explanations, show the short-run effect of each of the following taxes on the output and profits of a monopolist.
    2. An excise tax of 10 cents per unit of product.
    3. An excise tax of 10 percent of the price of the product.
    4. A corporate profit tax equal to 50 percent of net profits.
  1. (20 points) The Edgeworth Box Company is the only employer in the town of Yarmouth. Its supply schedule of labor is given by W = 40 + 1/4 q, where W is the wage in cents per hour and q is the number of manhours supplied per week. The company sells boxes in a competitive market. The value of the marginal product of labor is given by
    V = 100 – 1/2 q for values of q greater than zero.
  2. How many man-hours of labor will the company employ, and at what wage?
  3. Show diagramatically for part (a) first, the wage bill and second, the sum of monopoly profits and the return to factors of production other than labor.
  4. What will be the effect on employment of a legal minimum wage of 60 cents an hour? of 80 cents an hour?

This problem may be solved algebraically or graphically. The following table gives numerically some points on the schedules whose equations are given above:

Supply

Marginal Product

q (Man-hours)

W (cents) q (man-hours) W (cents)
1 40.25 1

99.5

2

40.50 2 99.0
3 40.75 3

98.5

4

41.00 4 98.0
etc.

etc.

Source: Duke University. David M. Rubenstein Rare Book & Manuscript Library. Economists’ Papers Archive. Albert Rees Papers, Box 1, Folder “Economics 300”.

Image Source: University of Chicago Photographic Archive, apf1-07002, Hanna Holborn Gray Special Collections Research Center, University of Chicago Library. Colorized by Economics in the Rear-View Mirror.

Categories
Business Cycles Distribution Economic History Exam Questions History of Economics Industrial Organization International Economics Johns Hopkins Labor Money and Banking Public Finance Public Utilities Statistics Theory

Johns Hopkins. General Written Exam for Economics PhD. 1956

 

One is struck by the relative weight of the history of economics in this four part (12 hours total) general examination for the PhD degree at Johns Hopkins in 1956. Also interesting to note just how many different areas are touched upon. Plenty of choice, but no place to hide.

________________________

Other General Exams from Johns Hopkins

________________________

GENERAL WRITTEN EXAMINATION FOR THE PH.D DEGREE
DEPARTMENT OF POLITICAL ECONOMY

*  *  *  *  *  *  *  *  *  *  *  *  *  *  *

PART I
June 4, 1956, 9-12 a.m.

Answer two questions, one from each group.

Group I.
  1. Write an essay on the theory of capital. It should include a discussion of the place of capital theory in economic analysis: for what purposes, if any, we need such a theory, Do not omit theories or issues which were important in the history of doctrines, even if you should regard them as irrelevant for modern analysis.
  2. Discuss and compare the capital theories of Böhm-Bawerk, Wicksell, and Hayek.
  3. Write an essay on the theory of income distribution. Organize it carefully, as if it were designed for an article in the Encyclopedia of the Social Sciences. Include discussions of alternative theories such as imputation theories, residual theories, surplus value theories, etc.
Group II.
  1. The following statements attempt to show that marginal productivity theory is inconsistent with factual observation. Accepting the stated facts as given, discuss whether they call for the rejection or major modification of the theory. If so, how? If not, why not?
    1. “In the most important industries in the United States wage rates are set by collective bargaining and are largely determined by the bargaining strength of the parties. Marginal productivity of labor is neither calculated nor mentioned in the process.”
    2. “In many industries competition among employers for workers is so limited that most firms are able to pay less than the marginal productivity of labor.”
    3. “Workers in some trades — say, carpenters or bricklayers — work essentially the same way as their predecessors did fifty years ago; yet their real wages have increased greatly, probably not less than in occupations where productivity has improved considerably over the years.”
  2. The determination of first-class and second-class passenger fares for transatlantic ocean transportation involves problems of (a) joint or related cost, (b) related demand, and (c) discriminatory pricing. Discuss first in what ways these three phenomena are involved here; then formulate a research project to obtain the factual information required for an evaluation of the cost relationships and demand relationships prevailing in the case of two-class passenger ships; and finally state the criteria for judging whether the actual rate differential implies conscious discrimination in favor of first-class passengers, conscious discrimination against first-class passengers, wrong calculation and faulty reasoning on the part of the shipping lines, or any other reason which you may propose.

*  *  *  *  *  *  *  *  *  *  *  *  *  *  *

PART II
June 4, 1956, 2-5 p.m.

Answer three questions, at least one from each group.

Group I.
  1. There is a running debate on the question whether trade unions are labor monopolies. This debate obviously turns on the meaning of monopoly and on what effects union have had on their members’ wages, output, and conditions of work. Give both sides of the argument.
  2. Write an essay on the demand for labor.
  3. Write down everything you know about the incidence of unemployment among various classes of workers and about the fluctuations of unemployment over time. Discuss some of the problems of developing a workable concept of unemployment. Indicate whether the statistical behavior of unemployment throws any light on its causation.
Group II.
  1. What is a “public utility”? According to accepted regulatory principles, how are the “proper” net earnings of a utility company determined? And, finally, what factors are considered in setting an “appropriate” rate structure?
  2. What is the major purpose of the Sherman Anti-Trust Act of 1890? What are some of the more significant problems in determining what constitutes “restraint of trade”? What tests would you apply? Why?
  3. Analyze the economic effects of a corporate income tax. Be as comprehensive as you can.
  4. What are flexible agricultural price supports? Explain how they are determined and applied. Evaluate their use in the light of reasonable alternatives.

*  *  *  *  *  *  *  *  *  *  *  *  *  *  *

PART III
June 5, 1956, 9-12 a.m.

Answer three questions, one from each group.

Group I.
  1. Describe briefly Schumpeter’s theory of economic development, and comment upon the possibility of testing it empirically.
  2. Describe briefly Keynes’ general theory of employment, interest and money; state its assumptions, structure, and conclusions; and evaluate it critically in the light of more recent theoretical and empirical findings.
Group II.
  1. What characteristics of economic cycles would you consider important in a statistical study of business cycles?
  2. In the study of long-term trends, what criteria would you use in constructing index numbers of production?
  3. What measures of economic growth of nations would you us? Consider carefully the various characteristics that you would deem indispensable in measurements of this sort.
Group III.
  1. Give a brief definition, explanation and illustration for each of the following:
    1. variance;
    2. confidence interval;
    3. coefficient of regression;
    4. coefficient of correlation;
    5. coefficient of determination;
    6. regression line.

[Note: Indicate where you have confined yourself to simple, linear correlation.]

  1. Write an essay on statistical inference by means of the following three techniques:
    1. chi square;
    2. analysis of variance;
    3. multiple regression.

Indicate the types of problem in which they are used, and how each type of problem is handled.

*  *  *  *  *  *  *  *  *  *  *  *  *  *  *

PART IV
June 5, 1956, 2-5 p.m.

Answer four questions, one from each group.

Group I.
  1. Political arithmetic is a term that is applied to certain writings that appeared from roughly 1675 to 1800. What gave rise to such writings? What were the contributions of the different members of the “group”? Why should Political Arithmetic be given a terminal date?
  2. Discuss Quesnay’s Tableau Économique, Do you see in it anything of significance for the subsequent development of economic theory?
  3. Present arguments for the contention that J. B. Say was far more than “a mere disciple of Adam Smith”.
Group II.
  1. Discuss the relations between the English economic literature of the first half of the 19th century and the events, conditions, and general ideas of that time.
  2. Select three episodes in American economic history, and use your knowledge of economic theory to explain them.
Group III.
  1. Analyze the economic effects of a large Federal debt. Be as comprehensive as you can.
  2. At one time or another each of the following has been proposed as the proper objective or goal of monetary policy: (1) The stabilization of the quantity of money; (2) The maintenance of a constant level of prices; (3) The maintenance of full employment.
    Explain for each policy objective (a) what it means, that is, exactly what in “operational” terms might be maintained or stabilized; (b) how the objective could be achieved, that is, what techniques could be used to achieve it; and (a) the difficulties with or objections to the proposal.
  3. Irving Fisher and others have proposed that all bank be required to hold 100% reserves against their deposits. This was designed to prevent bank failures and, more important, to eliminate the perverse tendency of money to contract in recessions and expand in booms.
    Explain whether the proposal would have the effects claimed for it, and if so, why, and discuss what other effects it might have.
Group IV.
  1. Discuss the “law of comparative advantage” in international trade.
  2. Discuss “currency convertibility”.
  3. Discuss the “transfer problem”.
  4. Discuss the “optimum tariff”.
  5. Discuss the “foreign-trade multiplier”.
  6. Discuss alternative concepts of the “terms of trade”.
  7. Discuss the “effects of devaluation upon the balance of trade”.

*  *  *  *  *  *  *  *  *  *  *  *  *  *  *

Source: Johns Hopkins University. Eisenhower Library. Ferdinand Hamburger, Jr. Archives. Department of Political Economy Series 5/6.  Box No. 6/1. Folder: “Comprehensive Exams for Ph.D. in Political Economy, 1947-1965”.

Image Source: Fritz Machlup in an economics seminar. Evsey Domar visible sitting third from the speaker on his right hand side. Johns Hopkins University Yearbook, Hullabaloo 1956, p. 15.

Categories
Exam Questions Harvard Theory

Harvard. Second year economic theory. Readings and exams. Leontief, 1960-1961

 

 

The following Harvard course outline with reading assignments and semester final exams are from the year 1960-61. Wassily Leontief taught the second graduate course in economic theory.

I have highlighted in blue boldface additions to the reading assignments in the 1960-61 course when compared to the 1956-57 version of the same course. Items omitted are listed at the end of the post.

Comparing the structure of the mid-year and year-end exams, I would conjecture that one or more of Koopmans’ Three Essays on the State of Economic Science was assigned for the first term’s reading period, though the title does not appear on the printed reading list for the course.

__________________________

Wassily Leontief

HARVARD UNIVERSITY
Ec. 202a
ECONOMIC THEORY
Fall Term 1960-61

The following outline covers the first semester of the two semester course.

I.     Analysis of Production and the Theory of a Firm:

  1. Costs; total, average, marginal.
    Theory of the multiple plant firm.
    Revenue; total, average, marginal.
    Long and short run analysis
    Supply under competitive and monopolistic conditions.
  2. Production function, marginal productivity, increasing and decreasing returns.
    Stocks and flows.
    Joint products.
    Demand for factors of production.
    Discontinuous relationships and non-marginal analysis (Linear Programming).
    Internal and external economies.

Reading assignments:

Oscar Lange, “The Scope and Method of Economics,” Review of Economic Studies, Vol. XIII, (1), 1945-46, pp. 19-32.

H. Simon, “Theories of Decision Making in Economics,” American Economic Review, June 1959.

E. A. G. Robinson, Structure of Competitive Industry, Chs. II, VII, VIII, pp. 14-35, 107-133.

R. C. Heimer, Management for Engineers, Chs. 3-17.

K. E. Boulding, Economic Analysis, (revised edition, 1948) Chapters 20-26, 31, and 32; or (3rded., 1955) Chapters 23-29, 34, and 35.

E. H. Chamberlin, “Proportionality, Divisibility, and Economies of Scale,”Quarterly Journal of Economics, February, 1948, pp. 229-262.

K. E. Boulding, “The Theory of the Firm in the Last Ten Years,” The American Economic Review, Vol. XXXII, No. 4, December 1942, pp. 791-802.

A. Lerner, Economics of Control, Chs. 15, 16, 17, pp. 174-211.

W. W. Cooper, “A Proposal for Extending the Theory of the Firm,” Quarterly Journal of Economics, February 1951, pp. 87-109.

R. Solow, “Technical Change and the Aggregate Production Function,” Review of Economics and Statistics, August 1957.

Robert Dorfman, “Mathematical or ‘Linear’ Programming,” American Economic Review, December 1953, pp. 797-825.

Dorfman, Samuelson and Solow, Linear Programming and Economic Analysis, Ch. 2.

H. M. Wagner, “The Simplex Method for Beginners,” Operations Research, March-April 1958.

R. Dorfman, “Operations Research,” American Economic Review, September 1960, pp. 575-623.

G. Katona, “Business Expectations in the Framework of Psychological Economics,” in M. J. Bowman, ed., Expectations, Uncertainty and Business Behavior.

II.    Theory of the Household:

  1. Theory of utility and indifference lines analysis.
    Individual demand, prices and income.
    Dependent and independent, competing and complementary, superior and inferior goods.
  2. Measurability of utility.
    “Marginal utility of money,” Consumer surplus.
    Interpersonal interdependence in consumers’ behavior.
    Economic theory of index numbers.
    Choices involving risk.

Reading assignments:

J. Hicks, Value and Capital, Part I, Chs. I-III, pp. 1-54.

K. E. Boulding, Economic Analysis, (Revised edition, 1948) Chapters 33, 34; or (3rd ed., 1955), Chapter 36 and 37.

Duesenberry, Income, Saving and the Theory of Consumer Behavior, Chapters I-III, pp. 1-46.

Modigliania and Brumberg, “Utility analysis and the Consumption Function,” in Kurihara, Post Keynesian Economics.

S. S. Stevens, “Measurement, Psychophysics and Utility,” in Churchman and Ratoosh (ed.) Measurement: Definitions and Theories, pp. 18-63.

A. A. Alchian, “The Meaning of Utility Measurement,” American Economic Review, March 1953, pp. 26-50.

D. Ellsberg, “Classic and Current Notions of ‘Measurable Utility’,” Economic Journal, September 1954.

H. Simon, Models of Man, Part IV, pp. 196-206.

III. Theory of Market Relationships:

  1. Pure competition; individual and market supply and demand curves.
    Stability of market equilibrium, statics and dynamics.
    Monopoly and price discrimination.
  2. Monopolistic competition.
    Duopoly, oligopoly, bilateral monopoly, etc.
    “Theory of games.”

Reading assignments:

A. Marshall, Principles of Economics, Book V, Chs. III, V.

E. H. Chamberlin, The Theory of Monopolistic Competition, Chs. II, III, IV, and V.

Joan Robinson, The Economics of Imperfect Competition, Chs. 15 and 16.

Robert Triffin, Monopolistic Competition and the General Equilibrium Theory, Chs. I and II.

William Fellner, Competition Among the Few, Chs. II-V.

W. H. Nicholls, Imperfect Competition within Agricultural Industries, Ch. 18.

F. Modigliani, “New Developments on the Oligopoly Front,” JPE,  June 1958.

Leonid Hurwicz, “The Theory of Economic Behavior,” American Economic Review, December, 1945, pp. 909-925.

D. Ellsberg, “The Theory of the Reluctant Duelist,” American Economic Review, December 1956.

T. C. Schelling, “An Essay on Bargaining,” American Economic Review, June 1956.

IV.  General equilibrium theory:

  1. Basic Concepts of a General Equilibrium Theory.
    Data and variables. Price system and general interdependence. Linear model of a general equilibrium system.
  2. Theory of Rent and Factor Prices

Reading assignments:

O. Lange, The Economic Theory of Socialism, pp. 65-72.

Cassel, The Theory of Social Economy, Vol. I, Ch. IV, pp. 134-155.

R. G. D. Allen, Mathematical Economics, pp. 314-325.

E. H. Phelps Brown, Framework of the Pricing System, in particular chapters dealing with general equilibrium theory.

T. W. Schultz, Agriculture in an Unstable Economy, Ch. I, pp. 60-70; Ch. IV, pp. 128-137.

R. S. Eckaus, “The Factor Proportion Problem in Underdeveloped Areas,” The American Economic Review, September 1955, pp. 539-565.

N. Georgescu-Roegen, “Economic Theory and Agrarian Economics,” Oxford Economic Papers, February 1960, pp. 1-40.

___________________________

Mid-year Examination
1960-1961 (Jan. 1961)

HARVARD UNIVERSITY
1960-1961
ECONOMICS 202

PLEASE WRITE LEGIBLY

Answer one question from each group, four questions in all.

GROUP I

  1. Demonstrate that the assumption that the marginal utility of one of the goods purchased by a consumer is constant is more restrictive than the assumption that its utility is independent of the quantity of any other good.
    How could the knowledge of the constancy of its marginal utility help to assess the effect of an income tax on the demand for the good in question?

GROUP II

  1. Amount Needed Per Unit of Activity Factor Supply
    Activity 1 Activity 2 Activity 3

    Factor 1

    6 1 2 12
    Factor 2 2 2 1

    10

    Factor 3

    1 5 20

    200

    Market Value Per Unit of Output

    15 3 8

    A firm with a fixed supply of three factors has three possible activities, each of which produces a different product selling for a different price. The factor requirements, factor supplies and the product prices are given in the table above. Find the level of activities, including disposal activities, which maximize the firm’s revenue.
    Supplemental information which can be used to shorten computation: In the solution of the “dual”, only factor 1 turns out to have a positive imputed price.

  2. A farmer has fixed amounts of two different kinds of land. He can grow two kinds of product, the prices of which are given. The only other input is labor. Its total available amount is also fixed. The amount of land and of labor required per bushel of each one of the two crops on each type of land is known.
    Set up the linear programming problem which the farmer would have to solve to maximize the value of his output.

GROUP III

4. (a) Discuss the differences and similarities of the following types of analysis:

      1. The derivation of a household’s demand curve for a commodity.
      2. The derivation of a firm’s demand curve for a factor of production.

4. (b) Demonstrate that,

        1. A household can have a positively sloping demand curve for the commodities it buys.
        2. A firm cannot have a positively sloping demand curve for any of the factors of production it buys, if it sells its product in a perfectly competitive market.
  1. A self-sufficient farmer lives on produce that he grows himself under conditions of decreasing average returns. The length of his working time can be explained in terms of a utility maximizing choice between agricultural produce and leisure.
    Among the (real) wage rates which could induce him to quit farming and become a hired worker, one necessarily must be lower than any other. If this minimum wage rate were actually offered to him, and he became a hired worker would the length of his working time a) remain the same b) become shorter or c) become longer than it was when he gained his livelihood as a self-sufficient farmer?

GROUP IV

  1. What is the principal contribution of the theoretical approach described in Koopman’s State of Economic Science?
    Write a critical essay on methodology, rather than substance, except where a discussion of the latter is necessary to a discussion of the former.

Source: Harvard University Archives. Social Sciences. Final Examinations, January 1961. (HUC 7000.28), Vol. 131 of 284.

___________________________

Wassily Leontief

ECONOMICS 202b
ECONOMIC THEORY
Spring Term, 1960-61

V.  Economics of Welfare

  1. Individual maximum and social welfare.
  2. Efficiency and distributive justice.
  3. Efficiency of the purely competitive system.
    Monopoly and economic welfare.
    External economies.
  4. Pricing and allocation for public enterprise.

READING ASSIGNMENTS:

J. Hicks, “The Foundation of Welfare Economics,” Economic Journal, December 1939, pp. 696-712.

Meade and Hitch, An Introduction to Economic Analysis and Policy, Part II, Chs. VI-VII, pp. 159-220.

Francis Bator, “The Anatomy of Market Failure,” Quarterly Journal of Economics, Vol. LXXII, August, 1958, pp. 351-379.

T. Scitovsky, “The State of Welfare Economics,” The American Economic Review, Vol. XLI, June 1951, pp. 303-315.

J. De Graaf, Theoretical Welfare Economics.

Mishan, E. J., “A Survey of Welfare Economics, 1939-1959,” The Economic Journal, Vol. LXX, No. 278, June, 1960, pp. 197-265.

VI. Capital and Interest

  1. Stock and Flow Concepts.
    Productivity of Capital.

    Period of production and “turnover” of capital.
  2. Theory of saving.
    Risk and uncertainty.
  3. Partial equilibrium theory of interest.

READING ASSIGNMENTS:

Robert Eisner, “Interview and Other Survey Techniques and the Study of Investment,” in Problems of Capital Formation, Studies in Income and Wealth, Vol. 19, National Bureau of Economic Research 1957, pp. 513-583. 

Irving Fisher, The Theory of Interest, Chs. VII, VIII, IX, X, XI, XVI, XVII, and XVIII. 1930.

Hirschleifer, “On the Theory of Optimal Investment Decision,” Journal of Political Economy, August, 1958.

Readings in the Theory of Income Distribution (Blakiston, 1946)

F. Knight, “Capital and Interest,” pp. 384-417.
Keynes, “The Theory of the Rate of Interest,” pp. 418-424.
D. H. Robertson, “Mr. Keynes and the Rate of Interest,” pp. 425-460.

Friedrich & Vera Lutz, The Theory of Investment of the Firm, 1951.

Joel Dean, Capital Budgeting, 1951, Chs. VI, VII.

Eckstein, “Investment Criteria for Economic development and Intertemporal Welfare Economics,” Quarterly Journal of Economics, Feb., 1957.

VII: Principles of Dynamics

  1. Change over time.
    Period analysis.
    Continuous change
  2. Dynamic stability and instability.

READING ASSIGNMENTS:

W. Baumol, Economic Dynamics, Chs. I-VII, pp. 1-136.

P. Samuelson, “Dynamics, Statics and Stationary State,” The Review of Economics and Statistics, February 1943, pp. 58-68 (also reprinted in Readings in Economic Analysis, Vol. 1, edited by N. V. Clemens).

K. J. Arrow, “Toward a Theory of Price Adjustment,” in The Allocation of Economic Resources, pp. 41-51, Stanford, California, 1959.

Erik Lindahl, Introduction to the Study of Dynamic Theory, pp. 21-73 in Studies in the Theory of Money and Capital.

Dorfman, Samuelson, Solow, Linear Programming and Economic Analysis, pp. 265-281.

VIII: Economic Development and Accumulation of Capital

  1. Dynamic interrelation of income, investment and the rate of interest.
  2. Linear theory of economic development.
    Non-linear theory of economic development.

READING ASSIGNMENTS:

Bresciani-Turoni, “The Theory of Saving,” Economica; Part I, Feb. 1936, pp. 1-23; Part II, May 1936, pp. 162-181.

Schelling, “Capital Growth and Equilibrium,” American Economic Review, Dec. 1947, pp. 864-876.

Harrod, “An Essay in Dynamic Theory,” Economic Journal, March 1939, pp. 14-33.

Stern, “Capital Requirements in Progressive Economies,” Economica, August 1945, pp. 163-171.

Robert M. Solow, “A Contribution to the Theory of Economic Growth,” Quarterly Journal of Economics, Vol. LXX, February, 1956, pp. 65-94.

Arthur Smithies, “Productivity, Real Wages and Economic Growth,” Quarterly Journal of Economics, May, 1960, pp. 189-205.

Also, Baumol, see above under VII.

IX: Keynesian Theory and Problems

  1. Over-all outlines of the General Theory.
    Wage and price “stickiness.”
    Demand for money.
  2. Saving and “oversaving.”
    Multiplier principle.

READING ASSIGNMENTS:

A. P. Lerner, The Economics of Control, Chs. 21, 22, and 25.

A. P. Lerner, “The Essential Properties of Interest and Money,” Quarterly Journal of Economics, May 1952, pp. 172-93.

J. M. Keynes, General Theory of Employment, Interest and Money, Chs. 1, 2, 8, and 18.

G. Haberler, Prosperity and Depression, Ch. 8.

Modigliani, “Liquidity Preference and the Theory of Interest and Money,” Readings in Monetary Theory.

Hicks, “A Rehabilitation of ‘Classical’ Economics?” Economic Journal, June, 1957.

Reading Period Assignment (spring):

Trygve Haavelmo, A Study in the Theory of Investment, Chicago, 1960.

OR

F. and V. Lutz, The Theory of Investment of the Firm, Princeton, 1951.

Source:  Harvard University Archives, Syllabi, course outlines and reading lists in Economics, 1895-2003. Box 8, Folder “Economics, 1960-1961 (2 of 2).

___________________________

Year-end Examination
1960-1961 (June 1961)

HARVARD UNIVERSITY
1960-1961
ECONOMICS 202

PLEASE WRITE LEGIBLY

Answer one question from each of the four groups, four questions all together.

GROUP I

  1. Consider a two commodity, two consumer group economy run along socialist principles. The government fixes the quantities of A and B produced in any one year, fixes ruble incomes going to consumer groups I and II and also fixes ruble prices so that the total income distributed can exactly buy the amounts produced.
    Assume that the collective behavior of a consumer group can be described as one of reaching the highest of a set of group indifference curves under collective income and market constraints.
    (a) Using the box diagram technique, show what additional
    conditions prices must satisfy if the market is to be cleared.
    (b) assume that equilibrium is not established at official prices and that the State decides to ration the available amount of the short commodity between the two groups.The rationing is done in such a way that both groups get less than they wish to buy at official prices.  Show how one can explain the resulting equilibrium.
    (c) Starting from this equilibrium, will the two groups necessarily find some advantage in exchanging commodities on the black market? Will the black market equilibrium be better or worse (in terms of conventional welfare criteria) than that obtained when prices fixed by the government are chosen so as to clear the market?
  2. It has been established that the annual cost of distributing electricity in an Indian city would be 100,000 rupees in capital charges plus one rupee per kilowatt consumed. The following proposal is put to a vote in a city referendum: “To build the system, charge a price of one rupee per kilowatt and tax the public 100,000 rupees to cover capital charges. The proposal is unanimously rejected. The city fathers then undertake a market survey and find that the tax could be reduced to 50,000 rupees, the price increased to 1.5 rupee and all costs still be exactly covered. They adopt this second proposal without further consultation.
    Assuming a homogeneous population and equal taxation, can you derive from the above information a preferential ordering of the following alternatives in terms of social welfare:

(a) Charging 1 rupee and raising 100,000 in taxes.
(b) Charging 1.5 rupee and raising 50,000 in taxes.
(c) Not building the system at all.

GROUP II

  1. A profit maximizing enterprise possesses a fixed plant and uses as its variable inputs a raw material (fixed amount per unit of output) and labor. Its finished product is sold and the raw material is purchased on perfectly competitive markets. On the labor market, however, the enterprise is the only employer; the workers are not organized and thus compete with each other.
    What factual information would you require and what theoretical construction would you use to explain the level of that enterprise’s output if,

(a) labor is hired on the basis of straight hourly wages.
(b) labor is hired at a flat hourly wage for the first eight, and a 50% higher overtime hourly rate for two additional hours, the workers being free to choose whether they want to work eight or ten daily hours.
(c) labor is paid flat piecework rates.

To simplify the problem, you are permitted to assume that the preference functions (real income vs. leisure) of all workers are identical.

  1. The growth of a certain kind of tree requires λ man-hours for planting and entails no other costs. The volume of wood represented by a tree increases at a constant growth rate:
    V = EXP(rt). Two alternative assumptions are made with respect to the tree market:

(A) Trees are sold by volume at a price p per volumetric unit.
(B) Trees are sold by volume at a price p’ now depending on the length of the tree. Observing that length is related to age, traders use the formula
p’= α SQRT(t), where α is a constant and t the age of the tree.

(a) Given the amount L of man-hours available per year, describe a “full employment” production process that guarantees constant profits, year after year.
(b) Under each market assumption, discuss the empirical possibility and operational usefulness of measuring the capital stock and its marginal productivity.
(c) Money can be lent and borrowed without limits at an interest rate i, which is larger than r. At what age should the trees be cut under assumption (B) if the grower wishes to establish a stationary production process that maximizes his utility over time?

GROUP III

  1. A conventional partial equilibrium theory explains the prices and the quantities — produced and consumed — of all goods on the assumption that a supply and a demand curve is given for each market.
    In what sense can it be said that, from the point of view of a general equilibrium theory, at least some of such given demand and supply curves must be either incompatible with each other or redundant? In answering this question, please use equations, graphs, or both.

GROUP IV

  1. To what extent does Haavelmo or the Lutz’s — whomever you have chosen to read — rely on purely technological specifications and considerations in describing and analyzing the role of capital in the operations of an individual enterprise and of the economy as a whole? And to what extent do they use definitions and measurements which “engineers” would not employ in their professional work?
    Illustrate your answer by specific examples.

Source: Harvard University Archives. Social Sciences, Final Examinations, June 1961 (HUC 7000.28, Vol. 134 of 284).

___________________________

Reading assignments in the 1956-57 reading list that were dropped from the 1960-61 reading list:

I.     Analysis of Production and the Theory of a Firm:

E. H. MacNiece, Production, Forecasting, Planning and Control, 292 pp.

R. Frisch, “Alfred Marshall’s Theory of Value,” Quarterly Journal of Economics, Vol. LXIV, No. 4, November 1950, pp. 495-524.

National Bureau of Economic Research, Cost Behavior and Price Policy, Ch. VII, pp. 142-169; Appendix C, pp. 321-329.

A. G. Hart, Anticipations, Uncertainty and Dynamic Planning, reprinted 1951, 98 pp.

II.     Theory of the Household:

J. R. Hicks, A Revision of Demand Theory, Parts I and II, also the summary and conclusion.

G. Katona, Psychological Analysis of Economic Behavior, Part II, #1-7, pp. 63-149.

III. Theory of Market Relationships:

No titles dropped.

IV. General Equilibrium Theory:

No titles dropped.

V.  Economics of Welfare

Coase, “Note on Price and Output Policy,” Economic Journal, Vol. LV, April 1945, pp. 112-113.

Samuelson, “Evaluation of Real National Income,” Oxford Economic papers, Jan. 1950.

VI. Capital and Interest

Edward F. Denison, “Theoretical Aspects of Quality Change, Capital Consumption, and Net Capital Formation,” in Problems of Capital Formation, Studies in Income and Wealth, Vol. 19, National Bureau of Economic Research 1957, pp. 215-260.

John Rae, John, New Principles of Political Economy, 1834, Chs. I-V.

Irving Fisher, Nature of Capital and Income, Chs. I, IV, V, XIV, XVII, Macmillan, 1906.

VII: Principles of Dynamics

K. E. Boulding, A Reconstruction of Economics, Ch. I, pp. 3-26.

VIII: Economic Development and Accumulation of Capital

Pigou, Economic Progress in a Stable Economy,” Economica, August 1947, pp. 180-188.

A. Sweezy, “Secular Stagnation?” in Harris, Postwar Economic Problems, McGraw-Hill, New York, 1943, pp. 67-82.

Hansen, “Economic Progress and Declining Population Growth,” American Economic Review, March 1939, pp. 1-15.

IX: Keynesian Theory and Problems

No titles dropped.

cf. The earlier post for Economics 202 in 1956-57.

___________________________

Image Source:  Wassily Leontief in Radcliffe Yearbook 1964, p. 98. Colorized by Economics in the Rear-view Mirror.

Categories
Funny Business Theory

Funny business. Emily Dickinson explains the 2nd theorem of welfare economics. 2023

I just asked ChatGPT

« Explain the second theorem of welfare economics in the style of an Emily Dickinson death poem. »

[Spoiler alert: her death poems can be sung to the tune of The Yellow Rose of Texas]

The response:

The Second Theorem lies in wait,
Its logic cold and clear,
It whispers to the market’s fate,
And marks the end so near.

With mathematics sharp as a knife,
It cuts through trade and choice,
Revealing truths about our life,
In market’s still, small voice.

It states that perfect competition,
Is what leads to best result,
For welfare of each citizen,
And market’s every pulse.

Yet still the Theorem’s voice is kind,
For it is but a guide,
To lead us to a better mind,
And market where all thrive.

So heed the Second Theorem’s call,
And let its wisdom lead,
For perfect competition’s all,
That our market needs.

Image Source: Wikipedia.deDaguerreotype of Emily Dickinson, c. early 1847. It is presently located in Amherst College Archives & Special Collections. Colorized by Economics in the Rear-view Mirror.

 

Categories
Exam Questions Harvard Suggested Reading Syllabus Theory

Harvard. Economic Theory. Enrollment, Readings, Exams. Carver, 1901-1902.

 

Professor Frank W. Taussig began what was to turn into a two year leave of absence starting with the academic year 1901-02. The previous year, assistant professor Thomas Nixon Carver apparently took over Taussig’s “advanced” theory course sometime late in the academic year and continued to teach it in the latter’s absence.

This post continues our series of Harvard’s economic courses for 1901-02, providing a linked reading list for Carver’s economic theory course along with the semester exams for the year-long course.

Carver’s 1949 autobiography is available at the hathitrust.org web archive. He writes there (p. 132):

At the end of the year, 1900-1901, Professor Taussig’s health failed, probably as the result of some very hard and discouraging work he had done on the State Tax Commission. He therefore took a year’s leave of absence which was lengthened to two years. This necessitated a change in my program.

___________________________________

Course Announcement

For Undergraduates and Graduates
  1. Economic Theory. Mon., Wed., Fri., at 2.30. Asst. Professor [Thomas Nixon] Carver.

Course 2 is intended to acquaint the student with some of the later developments of economic thought, and at the same time to train him in the critical consideration of economic principles and the analysis of economic conditions. The exercises are accordingly conducted mainly by the discussion of selected passages from the important writers; and in this discussion the students are expected to take an active part. Lectures are given at intervals outlining the present condition of economic theory and some of the problems which call for theoretical solution. Theories of value, diminishing returns, rent, wages, interest, profits, the incidence of taxation, the value of money, international trade, and monopoly price, will be discussed. Marshall’s Principles of Economics, Böhm-Bawerk’s Positive Theory of Capital, Taussig’s Wages and Capital, and Clark’s Distribution of Wealth will be read and criticised.

Course 2 is open to students who have passed satisfactorily in Course 1.

Source: Harvard University Archives. Annual Announcement of the Faculty of Arts and Sciences, Division of History and Political Science comprising the Departments of History and Government and Economics (June 21, 1901).  Official Register of Harvard University 1901-1902. Box 1. Bound volume: Univ. Pub. N.S. 16. History, etc. pp. 36-37.

___________________________________

Course Enrollment

For Undergraduates and Graduates:—

[Economics] 2. Asst. Professor Carver. — Economic Theory.

Total 32: 5 Graduates, 6 Seniors, 17 Juniors, 2 Sophomores, 2 Others.

Source: Harvard University. Report of the President of Harvard College, 1901-1902, p. 77.

___________________________________

Course Readings

ECONOMICS 2.
1901-1902

General Reading. Prescribed.

Marshall. Principles of Economics.
Taussig. Wages and Capital.
Böhm-Bawerk. Positive Theory of Capital.
Clark. The Distribution of Wealth.

References for Collateral Reading. Starred references are prescribed.

I. VALUE.

  1. Adam Smith. Wealth of Nations. Book I. Chs. 5, 6, and 7.
  2. Ricardo. Pol. Econ. Chs. 1 and 4.
  3. Mill.    “        “     Book III. Chs. 1-6.
  4. Cairnes.     “        “     Part I.
  5. *Jevons. Theory of Pol. Econ. Chs. 2-4.
  6. Sidgwick. Pol. Econ. Book II. Ch. 2.
  7. Wieser. Natural Value.
  8. *Clark. Philosophy of Wealth. Ch. 5

II. DIMINISHING RETURNS.

  1. Senior. Pol. Econ. Pp. 81-86.
  2. *Commons. The Distribution of Wealth. Ch. 3. 

III. RENT.

  1. Adam Smith. Wealth of Nation. Book I. Ch. 2. Pts. 1-3.
  2. *Ricardo. Pol. Econ. Chs. 2 and 3.
  3. Sidgwick.   “       Book II. Ch. 7.
  4. Walker.      “       Pt. IV. Ch. 2.
  5. Walker. Land and its Rent.
  6. Hyde. The Concept of Price Determining Rent. Jour. Pol. Econ. V.6. p. 368.
  7. Fetter. The Passing of the Old Rent Concept. Q.J.E. Vol. XV. P. 416.

IV. CAPITAL

  1. Adam Smith. Wealth of Nations. Book II.
  2. Senior. Pol. Econ. P. 58-81.
  3. Mill.      “       “       Book I. Ch. 4-6.
  4. Roscher.       “       Book I. Ch. 1. Secs. 42-45.
  5. Cannan. Production and Distribution. Ch. 4.
  6. Jevons. Theory of Political Economy Ch. 7.
  7. Fisher. What is Capital? Economic Journal. Vol. VI. P. 509.
  8. Fetter. Recent Discussion of the Capital Concept. Q.J.E. Vol. XV. P. 1.
  9. *Carver. Clark’s Distribution of Wealth. Q.J.E., Aug. 1901. 

V. INTEREST.

  1. Adam Smith. Wealth of Nations. Book I. Ch. 9.
  2. Ricardo. Pol. Econ. Ch. 6.
  3. Sidgwick.      “        Book II. Ch. 6.
  4. *Carver. Abstinence and the Theory of Interest. Q.J.E, Vol. VIII. P. 40.
  5. Mixter. Theory of Saver’s Rent. Q.J.E. Vol. XIII. P. 345.

VI. WAGES.

  1. Adam Smith. Wealth of Nations. Book I. Ch. 8.
  2. *Ricardo. Pol. Econ. Ch. 5.
  3. Senior.   “       “      Pp. 141-180 and 200-216.
  4. Senior. Lectures. Pp. 1-62.
  5. Mill. Pol. Econ. Book II. Chs. 11, 12, 13, and 14.
  6. Cairnes. Pol. Econ. Part II. Chs. 1 and 2.
  7. Sidgwick.        “      Book II. Ch. 8.
  8. Walker. “       “      Part IV. Ch. 5.
  9. Hadley. Economics. Ch. 10.
  10. *Carver. Wages and the Theory of Value. Q.J.E. Vol. VIII, P. 377.

VII. PROFITS.

  1. Walker. Pol. Econ. Part IV. Ch. 4.
  2. Hobson. The Law of the Three Rents. Quar. Jour. Econ. Vol. V. P. 263.
  3. Clark. Insurance and Business Profits. Quar. Jour. Econ. Vol. VII. P. 40.
  4. *Hawley, F. B. in Quar. Jour. Econ. Vol. VII. P. 459; Vol. XV. Pp. 75 and 603.
  5. MacVane, in in Quar. Jour. Econ.,  Vol. II. P. 1.
  6. Haynes, in               “     “       “     Vol. IX, P. 409.

Source: Harvard University Archives. HUC 8522.2.1, Box 1 of 10 (Syllabi, course outlines and reading lists in Economics, 1895-2003). Folder: 1901-1902.

___________________________________

Mid-year examination, 1902
ECONOMICS 2

Discuss the following topics.

  1. The relation of utility to value.
  2. The price of commodities and the price of services.
  3. Various uses of the term “diminishing returns.”
  4. The law of diminishing returns as applied to each of the factors of production.
  5. Prime and supplementary cost: illustrate.
  6. Joint and composite demand and join and composite supply.
  7. Quasi rent.
  8. Real and nominal rent.
  9. Consumer’s rent.
  10. The equilibrium of demand and supply

Source: Harvard University Archives. Harvard University Mid-year Examinations, 1852-1943. Box 6, Bound volume: Examination Papers, Mid-Years, 1901-02.

___________________________________

Final examination, June 1902
ECONOMICS 2

  1. State some of the different meanings which have been given to the law of diminishing returns, and define the law as you think it ought to be.
  2. Can you apply the law of joint demand to the wages fund questions?
  3. What is meant by an elastic demand and how does it affect monopoly price.
  4. Discuss Clark’s distinction between capital and capital goods.
  5. Under what conditions would there be no rent, and how would these conditions affect the value of products?
  6. Explain Clark’s theory of Economic Causation.
  7. What is the source of interest?
  8. What is the relation of the standard of living to wages?

Source: Harvard University Archives. Harvard University, Examination Papers, 1873-1915. Box 6, Bound volume: Examination Papers, 1902-03. Papers Set for Final Examinations in History, Government, Economics, Philosophy, Education, Fine Arts, Architecture, Landscape Architecture, Music in Harvard College (June, 1902), p. 21.

___________________________________

Collection of Carver’s economic theory readings and exams,
1900/01 through 1902/03

Harvard. Core economic theory. Readings and Exams. Carver, 1900/01-1902/03